Jump to content

Wikipedia:Reference desk/Science

From Wikipedia, the free encyclopedia

This is an old revision of this page, as edited by 155.144.251.120 (talk) at 03:32, 14 March 2008 (→‎Gases "dissolved" in liquids; fish breathing). The present address (URL) is a permanent link to this revision, which may differ significantly from the current revision.

Welcome to the science section
of the Wikipedia reference desk.
Select a section:
Want a faster answer?

Main page: Help searching Wikipedia

   

How can I get my question answered?

  • Select the section of the desk that best fits the general topic of your question (see the navigation column to the right).
  • Post your question to only one section, providing a short header that gives the topic of your question.
  • Type '~~~~' (that is, four tilde characters) at the end – this signs and dates your contribution so we know who wrote what and when.
  • Don't post personal contact information – it will be removed. Any answers will be provided here.
  • Please be as specific as possible, and include all relevant context – the usefulness of answers may depend on the context.
  • Note:
    • We don't answer (and may remove) questions that require medical diagnosis or legal advice.
    • We don't answer requests for opinions, predictions or debate.
    • We don't do your homework for you, though we'll help you past the stuck point.
    • We don't conduct original research or provide a free source of ideas, but we'll help you find information you need.



How do I answer a question?

Main page: Wikipedia:Reference desk/Guidelines

  • The best answers address the question directly, and back up facts with wikilinks and links to sources. Do not edit others' comments and do not give any medical or legal advice.
See also:


March 8

sodium azide

wat are the exact steps in making this? do u mix the chemicals in water,ect. —Preceding unsigned comment added by Steweydewey (talkcontribs) 00:04, 8 March 2008 (UTC)[reply]

please read sodium azide first. Furmanj (talk) 01:09, 8 March 2008 (UTC)[reply]
Just a word to the wise. If you plan on actually making sodium azide, please be careful with it. Wisdom89 (T / C) 03:10, 8 March 2008 (UTC)[reply]

ok,ok but can i get measurments?? —Preceding unsigned comment added by 76.14.124.175 (talk) 05:56, 8 March 2008 (UTC)[reply]

Despite its common use in airbags, sodium azide appears to be a fairly toxic substance that requires careful handling. If your only resource for relevant information is Wikipedia, then you probably shouldn't be trying to make it. Gandalf61 (talk) 10:10, 8 March 2008 (UTC)[reply]
It's not just fairly toxic. It's highly toxic, and death can (and has) occurred with the ingestion of as little as a milligram. Seriously, don't mess with this stuff. – ClockworkSoul 19:27, 8 March 2008 (UTC)[reply]

The Massive Multiverse Question

I'm not a nuclear physicist in any way but there is something that I can't shake from my mind. The multiverse. Is the Multiverse purely a hypothetical concept or is it something that exists (or could exist) in reality? I understand that everytime I make a decision, whether to turn left or right for example, a new universe is born. This seems bonkers to me, billions of people spawning trillions of universes. Is this really possible? I guess my question is... Is the multiverse theory something that just fits the equations and balances out the maths or is it a true, physical possibility for the reality that we live in? Do the multiverses ever overlap? Could I ever meet myself? I have a feeling that this question is a bit of an argumentative mine field. Any help and explanation (in simple terms!) would be greatly appreciated. Kirk UK —Preceding unsigned comment added by 87.242.131.156 (talk) 00:22, 8 March 2008 (UTC)[reply]

what makes you think that this existence is so unique? Or even real? This might make it even worse, but i'll point you to uniqueness and well defined. I know thats somewhat obscure but keep in mind your frame of reference to reality Furmanj (talk) 01:16, 8 March 2008 (UTC)[reply]
The many-worlds interpretation is just one possible interpretation of quantum mechanics, most do not create other universes. Basically, at this point, it's purely hypothetical, but is favored among those who think the laws of physics are deterministic (i.e. you can't have a simultaneously "half-alive/half-dead cat", it's either 100% dead or 100% alive, depending on your universe). As for "overlap" or traveling to other universes, I believe that the answer currently is almost definitely not (they're referred to as "non-communicating" parallel universes), but until we know whether it's true or not I don't think it can be ruled out entirely. That brings us to whether it's real or not, and at this point we don't know, and it may be that we never know, but I don't think that will keep us from looking. Hope that helps! -- HiEv 01:57, 8 March 2008 (UTC)[reply]
An example I saw a while back at some talk (where I freely admit I was not paying much attention) was ripples in a pool of water. There is the "big bang" as a rock hits the smooth surface of the water. Each ripple emanating from the center is a "world" that exists on its own. The further form the center, the further along the timeline it progresses. Pick a point in time (a specific distance from the center) and watch it. The ripples go past one after the other. Each one is a separate "world" and each one can have completely conflicting choices. So, at a specific point in time you may choose left, you may choose right, you may not exist at all. It depends on which ripple you are talking about. That's about all I picked up from the talk. Perhaps someone else knows what the guy was talking about and can explain it better. -- kainaw 02:07, 8 March 2008 (UTC)[reply]
I guess you've seen our article on the Multiverse hypothesis? Julia Rossi (talk) 06:12, 8 March 2008 (UTC)[reply]
The idea that we are "creating" universes with every choice we make is really quite silly. I think this idea comes from people misunderstanding one way of viewing multiverses in that one could see the differences as differences in choices made. Of course, the infinite number of universes with no life and therefore no possibility of "choices" would still be ifferent from each other and so the real differences between universes has to do with the playout of random events (both microscopic and macroscopic), which then affect choice. — Ƶ§œš¹ [aɪm ˈfɻɛ̃ⁿdˡi] 09:28, 8 March 2008 (UTC)[reply]
Is the Multiverse purely a hypothetical concept or is it something that exists (or could exist) in reality?
In short, this question, and many like it, are unanswerable. If believing in a multiverse helps you cope with your day-to-day existence, then go ahead and believe in it. If not, then it's just superstitious clutter that you can happily forget about. Vranak (talk) 15:36, 11 March 2008 (UTC)[reply]

solar or wood what is best for hot water

Why fire wood water heater is better than solar in winter and rainy season —Preceding unsigned comment added by Joseph-d-alappat (talkcontribs) 03:05, 8 March 2008 (UTC)[reply]

Well, since you're discussing specifics, consider how the conditions (rainy or windy) can effect wood heat and solar power. Wood just burns, and solar power is most effective in direct high-angle sunlight. — Lomn 06:15, 8 March 2008 (UTC)[reply]

Siamese communication

I have a Siamese cat. Like every other Siamese I know, he has three states: sleeping, eating, and meowing. He has the typically large Siamese vocabulary and expresses himself almost constantly when people are in the room. Obviously, he's doing it to communicate something to us (he doesn't meow at the chair when I'm not there, for example). What I was wondering was - do Siamese cats owned by deaf people meow less? Do they learn that talking doesn't work and try communicating in other means? Besides idle curiosity, it might also shed light on feline intelligence. Matt Deres (talk) 03:28, 8 March 2008 (UTC)[reply]

I would test that theory some how. good question. But never underestimate animal intelegence —Preceding unsigned comment added by 76.14.124.175 (talk) 05:58, 8 March 2008 (UTC)[reply]

It's not just siamese cats that miaow a lot. One of mine, who is just some form of moggie (probably british shorthair tuxedo cat) likes to miaow a lot, and you can have conversations with him. I have no idea what I'm saying, but he seems to enjoy them. -mattbuck (Talk) 17:24, 8 March 2008 (UTC)[reply]
My parents have a cat (generic short hair black cat) that has a toy that is just feathers on the end of a stick. She now picks it up in her mouth, drags it to one of us, drops it, and meows until one of us plays with her. -- MacAddct  1984 (talk • contribs) 19:52, 8 March 2008 (UTC)[reply]
My cats meow when they want food (or are otherwise unhappy), but otherwise seem to talk to each other and not to me. However, there doesn't seem to be much variety in their conversation: it's mostly "where are you?" as best I can tell. --Tardis (talk) 00:02, 11 March 2008 (UTC)[reply]

Schrödinger Equation

Can someone please give me an example of the Schrödinger equation with each step, please? Thanks, Zrs 12 (talk) 04:39, 8 March 2008 (UTC)[reply]

I'm no scientist, but → Schrödinger equation. --~~MusicalConnoisseur~~ Got Classical? 06:57, 8 March 2008 (UTC)[reply]
If you wanted an example of a solution to Schrödingers equation for a specific system, then perhaps the easiest to understand is Particle in a box. You can find a list of Wikipedia articles with analytical solutions for other geometries here. SpinningSpark 14:15, 8 March 2008 (UTC)[reply]

How Halbach array works ?

is it necessary for all the magnets of a halbach array to be of same strength ? iwe know every magnet have two poles but in cace of halbach array, the law is violated ; what's the explanation of strange behaviour of halbach array ? —Preceding unsigned comment added by Shamiul (talkcontribs) 05:43, 8 March 2008 (UTC)[reply]

See our Halbach array article. It does not mention different strengths, so I would assume that, yes, the magnets in the array are all the same strength. We also have an article on the Halbach cylinder. Gandalf61 (talk) 09:50, 8 March 2008 (UTC)[reply]
The Magnet article explains that magnets can have any number of poles, except one, so the Halbach array does not violate anything. Refrigerator magnets are common examples of multipole magnets. --Heron (talk) 13:42, 8 March 2008 (UTC)[reply]

what mathematician (von neumann? erdos?) strongl argued for preemptive annihilation of Russia?

What famous mathematician campaigned during the cold war for a preemptive annihilation of russia via nuclear weapons, based on Game theory? Citation? —Preceding unsigned comment added by 79.122.0.69 (talk) 13:55, 8 March 2008 (UTC)[reply]

According to the von Nuemann article he was in favour of a pre-emptive strike to prevent the USSR developing nuclear weapons. However, Game Theory is not mentioned in the article. SpinningSpark 14:30, 8 March 2008 (UTC)[reply]
Yeah I'd go with Von Neumann. He was very hawkish, and remember that he died while the US still had a massive nuclear lead over Russia. --98.217.18.109 (talk) 14:57, 8 March 2008 (UTC)[reply]
As for a citation, Steven Heims says Von Neumann as an advocate of preemptive war at least around 1950 (when, as I note, it was still feasible, as the USSR only had a bomb or two ready to go), in his book John Von Neumann and Norbert Wiener: From Mathematics to the Technologies of Life and Death. --98.217.18.109 (talk) 15:58, 8 March 2008 (UTC)[reply]
It was definitely von Neumann, who had a special hatred of Communists due to his background. He's widely quoted as saying "If you say why not bomb them tomorrow, I say why not today? If you say today at 5 o'clock, I say why not one o'clock?" (refs). Note also that at the time there was a careful distinction made between preemptive war and preventive war, with the latter more extreme position being von Neumann's. Curtis LeMay had similar feelings, along with many other deeply fearful and pessimistic people of that scary new era. --Sean —Preceding unsigned comment added by 69.134.115.242 (talk) 17:21, 8 March 2008 (UTC)[reply]
Note that Von Neumann later changed his position. I can't remember the details, but I believe the talk I heard this in to be similar to this (which I can't get to work). Algebraist 17:48, 8 March 2008 (UTC)[reply]
Von Neumann's position changed to support of the MAD strategy as the USSR capabilities increased and first strike started to look more dangerous. SpinningSpark 19:21, 8 March 2008 (UTC)[reply]

radiation protection

i wnat to design a radiography department in a health care centre wat kind of radiation protection measures should i use and why should i use themRowin.r (talk) 14:34, 8 March 2008 (UTC)rowin.r[reply]

I've got to assume here that this is not a practical question (because no one would be given such a job who didn't know about how to do it), but is probably some sort of homework or assignment question. Look at your course materials; any answers we give will probably not be the ones your teacher is looking for. --98.217.18.109 (talk) 14:55, 8 March 2008 (UTC)[reply]
We have an article on radiation protection you might want to look at. Bear in mind that only some of the ideas in the article would be applicable to a radiology department. SpinningSpark 17:22, 8 March 2008 (UTC)[reply]

Red-eared sliders?

Hi. A friend had wanted me to ask, is it safe to put together two red-eared sliders of different age (eg. one a few months old and one three years old)? What about gender? How much space do they need, if you put two together? Thanks. ~AH1(TCU) 17:32, 8 March 2008 (UTC)[reply]

Quoth this site (used as a reference in our red-eared slider article) [1]:"A hatchling should never be added with adult turtles because of accidental or intentional injuries. Only turtles of a generally similar size with similar habitat requirements should be together in a confined enclosure. "
The site also has other information on keeping red-eared sliders, including this on space: "A guideline to determine this size used by many keepers, as a minimum, is 10 gallons of tank per each inch of shell length (refer to SCL for correct shell measurement). Therefore, a single adult RES will require anywhere between a 90 to a 120 gallon tank."

AlmostReadytoFly (talk) 19:26, 8 March 2008 (UTC)[reply]

Blueshifted stars?

Hi. What are all the known blueshifted stars (ones with a negative radial velocity)? This is not homework. Should we have an article such as "list of blueshifted stars" or "list of stars with a negative radial velocity" or "list of stars with decreasing distance"? Thanks. ~AH1(TCU) 17:37, 8 March 2008 (UTC)[reply]

That would be one humungous list, mostly filled with redlinks (does that mean the articles are receding from Wikipedia?). Clarityfiend (talk) 18:02, 8 March 2008 (UTC)[reply]
Exploration of the Universe (Abel, Morrison, Wolf) lists 16 amongst the nearby stars. As there are several blue-shifted galaxies out there, the total number of blue-shifted stars is going to be uncatalogable. However, if you would like the data from the tables in this book for an article, drop a note on my talk page and I will provide it. SpinningSpark 18:12, 8 March 2008 (UTC)[reply]
There are several blueshifted galaxies? Which are they and are they in the local group? Thanks. ~AH1(TCU) 18:34, 8 March 2008 (UTC)[reply]
According to AMW, the galaxies in the local group with negative radial velocity are; NGC6822, NGC185, NGC205, M32 (NGC221), IC1613, Andromeda galaxy (M31) and M33 (NGC598). As I said, if you want all the figures (velocities, positions etc) for an article, let me know. SpinningSpark 18:48, 8 March 2008 (UTC)[reply]

hi guys, I'm just someone else but can you tell me why it's not the case that half of all stars are blue-shifted? it's a toss-up between red and blue shifted, right? There aren't any that aren't shifted are there, by however little.... —Preceding unsigned comment added by 79.122.0.69 (talk) 18:36, 8 March 2008 (UTC)[reply]

Not so. The majority of the red shift is caused by the expanding universe and increases with distance dramatically, see Hubbles law. It is only a bit of nearby (local galaxies) local stuff that can be blue shifted and not completely swamped by the universal expansion. SpinningSpark 18:42, 8 March 2008 (UTC)[reply]
(edit conflict, my cordless mouse is so f***ed up and slow)Hi. Well, the universe is expanding, so especially the far-away stars and galaxies are moving towards us. Parts of the milky way movement are uneven. It's also likely that if two stars are moving in 3D space in the same direction that most of them will move away from us. Also, I was right that Andromeda and Triangulum are blueshifted! So in an expanding universe, blueshifted galaxies, even close-by ones, mean that they might merge with us, as in the case with Andromeda and Triangulum. Blueshifted stars will soon be redshifted anyway, because they will soon reach minimum distance from us and move away. Redshifted stars will likely continue to be redshifted as they continue to move away. Thanks. ~AH1(TCU) 18:47, 8 March 2008 (UTC)[reply]

Image search related question

Is there any algorithm that search image using image has been develop? For example I would like to search apple image so I'm using an image of apple to find the image that I want. If it has could you provide with some useful links? Thanks. roscoe_x (talk) 18:32, 8 March 2008 (UTC)[reply]

yes. what language are you using? —Preceding unsigned comment added by 79.122.0.69 (talk) 18:39, 8 March 2008 (UTC)[reply]

Wait, are you saying you want to search for other similar images based on a source image? If so, then I'd say no, or at least I'm not aware of any such search algorithm existing currently. The problem is that computers are really terrible at image recognition at this point, so a computer's idea of "similar" isn't much like a human's idea of "similar". I doubt any search using current technology would do exactly what you're looking for. There is Google Image Search which allows you to search for images using words, but that's not quite the same thing. -- HiEv 19:41, 8 March 2008 (UTC)[reply]
Note that to a computer a picture of an an apple is just something round-ish and red. It won't know it's a fruit and it won't know it's edible and it won't know that there are green ones too, or sliced ones, or anything else. It doesn't have the complex range of associations that a human would have. You'd probably as likely find a beach ball in response to your query as another apple. --98.217.18.109 (talk) 20:35, 8 March 2008 (UTC)[reply]
I use C, C++, and Java. But I don't intend to develop such a program. I just want to know the algorithm. Every object has pattern, the apple only a sample. The application might be used in finding finger-print, facial recognition, or maybe other useful things. roscoe_x (talk) 01:20, 9 March 2008 (UTC)[reply]
You should start with our article on computer vision. If you're looking for a single algorithm to recognize an apple, you're going to be disappointed; computers are dum. That said, here's something neat: take a Google Image Search like say, http://images.google.com/images?q=wikipedia, and add '&imgtype=face to your query: http://images.google.com/images?q=wikipedia&imgtype=face. Neat! Maybe if someone gives you a better answer than me, Google will hire you to implement imgtype=apple. :) --Sean —Preceding unsigned comment added by 69.134.115.242 (talk) 03:00, 9 March 2008 (UTC)[reply]
O well that's very cool too. I've check 10 pages and they all giving me correct results. And what amaze me is no mistake and the search result could gives different facial expressions. Do you know the basic? Are they using edge detection? I understand if its classified. Thanks. roscoe_x (talk) 13:37, 9 March 2008 (UTC)[reply]
One thing I would think computers could do at this point is find the same image, perhaps at a different scale or color depth. This alone might be useful, if you have a small, greyscale pic and want a large color version of the same thing. Perhaps different cropping, aspect ratios, mirror images, image formats, etc., could also be accommodated. Some of the same technology used to compare fingerprints could be used here. This technology could then be put to use to solve one of the greatest frustrations facing mankind, how to use a nude pic of a celebrity with their naughty bits blacked out to find the original. :-) StuRat (talk) 18:19, 9 March 2008 (UTC)[reply]

See OPTIMOL. This is a project out of Princeton that builds a set of images based on a couple of "seed" images of the object class of interest. Sancho 18:54, 9 March 2008 (UTC)[reply]

Characteristics of skin Types

I am trying to find the characteristics of the skin type of chinese skin and also black skin. I am learning about skin and the differences of there skin types, other than the colour I cant find anything else. Please help me do my college research. I have done european and asain. but the other two I cant find anywhere on the internet. Beechview (talk) 18:41, 8 March 2008 (UTC)[reply]

Hair follicles are part of the skin, so any differences in hair type would also technically be differences in skin type. Beyond that, I don't know. If you look hard enough for differences and can't find them, you could conclude that scientists may not be aware of any. --Allen (talk) 19:03, 8 March 2008 (UTC)[reply]
But even if there are no differences beyond skin color and hair texture, there's still a lot to be said on the subject of human skin color. You may also be asking too much of categories like "Chinese skin" and "black skin"... it's really all a big continuum, as the map at human skin color suggests, with a lot of variation at all scales. --Allen (talk) 19:12, 8 March 2008 (UTC)[reply]
FYI - If you've done Asian skin then you've already done "Chinese skin", since China is a part of Asia. -- HiEv 19:44, 8 March 2008 (UTC)[reply]
Asia is a large group of different people. The OP most likely meant a specific group, such as Japanese. Because he is referring to the skin, I am reminded of one of Bruce Lee's movies where the Japanese refer to the Chinese as "sick yellow men of China" - referring their skin tone. Of course, China itself has a wide variety of people in height, weight, and skin color due a long history of being invaded and, for the most part, absorbing the invaders into the collective whole of the country. -- kainaw 02:54, 9 March 2008 (UTC)[reply]
Hi. Here's a link if you want to know about the invasion of China and related matters during world war II (warning: potentially disturbing content). Thanks. ~AH1(TCU) 23:52, 9 March 2008 (UTC)[reply]

Adding salt to boiling what-have-you

If adding salt lowers the boiling temperature, why not just cook the stuff at that lower temperature (say 90 or 95 degrees or whatever it may be). What's so special about making the water boil around the potatoes? I think they'll soften just fine at 90 or so sans bubbles. ----Seans Potato Business 18:51, 8 March 2008 (UTC)[reply]

Adding salt doesn't lower the boiling point of water; indeed, it doesn't significantly affect the boiling point at all. (The boiling point will be elevated very slightly – less than one degree [2] – with the quantities of salt typically used in the kitchen.)
The addition of salt may, however, affect the flavour or texture of the cooked food. TenOfAllTrades(talk) 19:09, 8 March 2008 (UTC)[reply]
Adding salt elevates the boiling point (colligative), which means that the water boils at a higher temperature which means your spaghetti cooks faster.--Mmoneypenny (talk) 00:40, 9 March 2008 (UTC)[reply]
No, as Ten points out, the amount of salt added to, say, a pot of water to cook spaghetti in, is not enough to cause any appreciable degree of boiling point elevation. The idea that salt is used in cooking to manipulate the boiling point is simply a wide-spread misconception. The salt is added for taste. If you added enough salt to make a significant difference in boiling point, the food would be so salty it would be inedible. Any (marginal) increase in boiling point from the pinch of salt that's actually used in cooking is incidental and unimportant. - Nunh-huh 05:10, 9 March 2008 (UTC)[reply]
It is certainly true that adding salt to a pan of hot, but not quite boiling, water can suddenly produce a rolling boil. Of course, this may possibly be less to do with the salt in solution lowering the boiling point and more to do with the salt crystals providing nucleation sites for the steam to form. SpinningSpark 01:18, 9 March 2008 (UTC)[reply]
Adding salt is a lot to do with food colour too. Try purple sprouting brocoli in two pans one with one without salt: the difference is very striking. --BozMo talk 08:25, 9 March 2008 (UTC)[reply]
If the addition of salt produces a rolling boil, then the pot of water was actually already above the boiling point of water—at least at the bottom of the pot. Water in the bottom of the pot can become slightly superheated; this process is aided by two factors. First, if the pot is particularly smooth, there will be few nucleation sites around which bubbles can form. Second, if the water is fairly deep and/or is unstirred, hot water can be trapped at the bottom of the container; the weight of the water on top slightly elevates the pressure at the bottom of the pan and thereby also slightly elevates the boiling point.
Agitating the liquid in the pot (to bring hot water to the surface) or providing nucleation sites for bubble formation will both encourage sudden, rapid boiling. Adding salt can drive both processes. (Incidentally, this is one reason to use extreme care in heating mugs or cups of liquid in the microwave. An apparently not-quite-boiling cup of liquid can suddenly boil violently when disturbed: [3], YouTube video.) TenOfAllTrades(talk) 16:39, 9 March 2008 (UTC)[reply]


March 9

Animal behaviour

I've read that chimpanzees are pretty violent and often rape and murder one another, and that dolphins kill for fun, although they kill big fish and not other dolphins. Humans are obviously prety violent what with our murdering and our warfaring, so my question is this: discounting territorial disputes, does propensity for violence increase with intelligence among animals? I mean with average species intelligence, not individual cases. And I mean 'senseless' violence rather than mating/food etc related. Thanks 81.96.160.6 (talk) 04:54, 9 March 2008 (UTC)[reply]

"Senseless" violence in animals is not really senseless. Just because we don't completely understand the motivation of the animals, does not mean that there is not good biological reasons for it. With regards to chimps that kill and rape, it may be a dominance behaviour, used to establish roles in society. I think its difficult to generalize but among mammals increasing intelligence is generally associated with less violence, since more intelligent animals tend to develop strategies of co-operation and exist in complex social structures. These tend to defuse aggression, rather than stimulate it. Male mice, for example, will robustly attack any other male mouse that happens to wander into his territory. When another guy comes into your house, (I expect) you don't attack him on sight. Rockpocket 09:12, 9 March 2008 (UTC)[reply]
If he comes in through the window, uninvited, I sure do. :-) StuRat (talk) 18:01, 9 March 2008 (UTC)[reply]
Hi. Remember that chimps have a ⅓ murder rate, meaning that one in three of all chimps become murdered at some point in their lives. Also, sometimes bottlenose dolphins will bully, attack, and kill Dall's porpoises. It may be hard to believe, but that "smile" of a dolphin is actually a threat. Occasionally but very rarely, dolphins might attack humans as well. Chimps are seven times stronger than humans. Humans ususally don't go on these "senseless" attacks because we are generally more civilized and have a code of laws. In fact, some consider the cimps' fighting and killing of monkeys a form of politics. Hope this helps. Thanks. ~AH1(TCU) 14:36, 9 March 2008 (UTC)[reply]
It could be argued that "murder" or the killing of your own species or of other species is part of human behaviour. There are people that train in wrestling and forms of martial arts - it seems to be part of human nature to be capable of this, and in an animal system where it's survival of the fittest and you need to dominate to find a mate, it's part of nature. Humans have similar behaviour - we have sports, and many people do fight (even mock fight) and try to dominate other people physically - of course even psychologically, verbally and sexually. Rfwoolf (talk) 18:50, 9 March 2008 (UTC)[reply]

But the mouse does have motivation: it's territorial. I'm not saying it's right, but I mean really killing for fun, like dolphins apparently do. Where there is really no gain. The chimps thing works: social gain is really an intelligence related thing so killing for that sort of maybe demonstrates a link. It just dosn't seem that many "stupid" animals go round getting their kicks from murder. 81.96.160.6 (talk) 19:41, 9 March 2008 (UTC)[reply]

I'm not sure killing is ever without purpose. If nothing else, it provides practice for a time in the future when killing might be necessary for survival. For example, a well-fed cat will still catch mice, even if it has no interest in eating them. They even let the mouse go, then capture it again and again, in order to sharpen their skills. In the case of humans, hunting may be an important survival skill in some situations, so the desire to hunt does have a purpose. The ability to hunt other humans can also be an important survival skill in war, but the ability to turn this aggression off when not needed is far more important in this case, as a person who hunts other people during peacetime will soon found himself hunted. StuRat (talk) 15:39, 12 March 2008 (UTC)[reply]

DNA sequencing

It seems that before a DNA sequence is sequenced, it is cloned into a plasmid for amplification in bacterial cells, then the plasmid is cut for sequencing. Why can't PCR be used to amplify the DNA for sequencing? ----Seans Potato Business 08:58, 9 March 2008 (UTC)[reply]

You can sequence a PCR product also. The problem, though, is that the thousands of amplicons may contain slightly difference sequences due to PCR error which can result in a messier read. Cloned sequences are more likely to be homogeneous. If you want to do something with the DNA after you determine the sequence then you clone it first (since you can't be sure which of the many PCR amplicons you have sequenced), but if you just want to determine what you have amplified, then you can sequence the PCR product without cloning it first. Rockpocket 09:16, 9 March 2008 (UTC)[reply]
Generally speaking, a plasmid is not cut before sequencing. In fact, invariably the plasmid is purified and left intact before sequencing with a primer. However, as stated above, one can always sequence a PCR product. And just as a note, PCR and sequencing are fairly similar. There is an extension reaction that needs to take place before the gel fluorescence is read by automated sequencer. Wisdom89 (T / C) 14:38, 9 March 2008 (UTC)[reply]
Also note that some of the newer sequencing techniques don't require plasmid cloning. George M. Church's polony technique comes to mind, as do some of the nanopore sequencing techniques. Most of these aren't being used commercially yet, though.-- 128.104.112.85 (talk) 19:56, 11 March 2008 (UTC)[reply]

Energy

Which of these..(radiation, conduction, convection)...does heat from the sun reach the earth?? —Preceding unsigned comment added by Cassiesirevatarax (talkcontribs) 10:03, 9 March 2008 (UTC)[reply]

The answer is in the first sentence of the Sunlight article. --Heron (talk) 10:52, 9 March 2008 (UTC)[reply]

Firing on all pistons

What engines can be configured to fire on less than all pistons? ----Seans Potato Business 10:21, 9 March 2008 (UTC)[reply]

See Variable displacement and Active Fuel Management. --hydnjo talk 11:52, 9 March 2008 (UTC)[reply]
Your internal combustion engine can often work on less than the full number of cylinders, when a spark plug or lead goes bung. Graeme Bartlett (talk) 13:21, 9 March 2008 (UTC)[reply]
Would this be damaging to the engine? ----Seans Potato Business 22:28, 9 March 2008 (UTC)[reply]
No, but allowing an unburned fuel-air mixture to reach the catalytic converter will cause the converter to overheat and sometimes igniite fires in the surrounding materials. Backfires may also occur in the exhaust system and these can blow apart weak points.
Atlant (talk) 16:32, 10 March 2008 (UTC)[reply]
It's not generally a good thing efficiency-wise, because those cylinders are still getting dragged along for the ride while they're not providing useful power. Still, displacement on demand is being actively used and developed by Chrysler among others. Forced induction is a competing approach, from the other direction- this lets you have a small engine that "gets bigger" under load, rather than a big engine that gets smaller under light load. Friday (talk) 16:38, 10 March 2008 (UTC)[reply]

Which Australian native frog?

Which Australian native frog in the East coast region of NSW makes a sound like a piece of wood going clunk or "clock" against something? It's small and brown and turns up in metropolitan Sydney. Julia Rossi (talk) 10:32, 9 March 2008 (UTC)[reply]

Could it be the pobblebonk? Adambrowne666 (talk) 12:20, 9 March 2008 (UTC)[reply]

Thanks Adambrowne, pobblebonk gives the eastern banjo frog so I'm assuming when they say it sounds "like a plucked banjo string" means in an non-tuneful, "pock" way for percussive effect. Any banjo players confirm that? Julia Rossi (talk) 21:28, 9 March 2008 (UTC)[reply]

Quantum Mechanics: Operator and Eigenvalue

For a given wave function of a particle at position , the momentum of the particle is the eigenvalue of (1)

(1)

where

(2)

For example, if the wave function of a particle is

(3)

, the corresponding momentum will be

(4)
(5)
(6)
(7)

Therefore, the momentum of the particle (3) is . But does it make sense to say the coordinate of the particle (3) is the eigenvalue of (8)? It seems that we will always get if we replace (3), or any other wave function, into (8)!

(8)

Justin545 (talk) 11:05, 9 March 2008 (UTC)[reply]

I think the problem you are running into here is Heisenbergs uncertainty principle which states that there is an unavoidable minimum uncertainty in the product of the momentum and position observables;

If you claim to know with certainty that the momentum of the particle is then the position must, of necessity, be completely indeterminate. There is a similar relationship between other pairs of observables, such as Energy and Time. SpinningSpark 13:17, 9 March 2008 (UTC)[reply]

Uncertainty principle aside, most wavefunctions are not eigenvectors of most Hermitian operators. In fact no proper (normalizable) wavefunction satisfies for any p. An equation like is not meant to be solved for a as a function of ψ, it's meant to be solved for ψ as a function of a. Most wave functions won't be in the solution set, but they'll be expressible as a sum of elements of the solution set. If you like you can think of Hermitian operators like as an odd way of specifying an orthogonal basis with a real number attached to each basis vector. -- BenRG (talk) 15:50, 9 March 2008 (UTC)[reply]
It turns out what I did was just replace the solution, or the basis, (3) into (1) according to the reply. I also forgot the position of a particle is uncertain, it is good to recall Heisenberg's uncertainty principle. I just did some ridiculous generalization and thought that the operator can be used as (8) which is similar to (1) :p But it seems the operator is useless except it is only used to calculate the mean value - Justin545 (talk) 03:37, 10 March 2008 (UTC)[reply]

how do astronauts refer to spacesuits

The wiki space suit article is great, but I wonder if astronauts call them 'space suits' also, or use an abbreviation or colloqialism to refer to them. Can anyone enlighten me? Thanks Adambrowne666 (talk) 12:17, 9 March 2008 (UTC)[reply]

Normally, they'd just be space suits, although there are a number of different types, so astronauts probably just call them by their name - the Advance Crew Escape System Pressure Suit (commonly called the ACES) and the Extravehicular Mobility Unit (the EMU) are the two used on most American missions, for example. Russian space suits (Скафандр) are all named after types of bird: the Yastreb translates to "Hawk" for example. Laïka 16:12, 9 March 2008 (UTC)[reply]
Thanks, Laika - I like your space-themed name, btw. Adambrowne666 (talk) 22:48, 9 March 2008 (UTC)[reply]

Chemistry

Give an octahedral complex containing an optically active ligand? —Preceding unsigned comment added by Minmoy mld (talkcontribs)

Sounding like homework: so the complex may or may not be optically active, but the ligand is? Many of the biomolecules that incorperate a metal may be like this. cyanocobalamin is one example. Graeme Bartlett (talk) 13:18, 9 March 2008 (UTC)[reply]

A gamma ray burst extinction scenario

(Bolded questions) I have been reading up on the GRB articles for fun. Not so fun are the theories about some random GRB making us all extinct, in effect at least. My question is not about what is a good idea or not to help prevent the earth from taking damage, but if it is possible to build a kind of screen that could absorb the rays on earth's behalf, shielding earth, and most importantly what material would suffice? The disc, made of whatever we would harvest from other planets, would have a diameter a bit greater than that of earth. It's placed at a good distance from earth (in the hundreds of thousands, or millions of kilometers), and to make matters worse, would need to somehow follow earth's orbit around the sun. Assume of course, that we know exactly where the GRB is likely to come from (which isn't entirely unrealistic). Are there, for the sake of adding meat to this question, any serious discussions within scientific groups how to best protect earth against a GRB? 81.93.102.185 (talk) 13:17, 9 March 2008 (UTC)[reply]

The "screen" would probably need to be several metres thick to provide any useful protection. But let's suppose, for simplicity, we make a disk the same radius as the Earth but just 1 metre thick. This would require the same amount of matter as a sphere about 60 km in diameter - something about the same size as a small moon of Jupiter. Now remember that the hypothetical asteroid that may have wiped out the dinosaurs is estimated to have been only about 10 km in diameter. I think the risks caused by having such a massive object anywhere near the Earth's orbit would be much greater than the GRB risk that it was protecting us from. Gandalf61 (talk) 15:09, 9 March 2008 (UTC)[reply]
Hi. Well, it might be possible if you could somehow put the object in free-fall around the Earth. Now, an object that big would weigh roughly 5x1014 tons if I calculated correctly, so it would be pretty hard for it to get off the Earth in the first place, unless you assemble it in space, which would take decades and trillions of dollars for something that probably won't happen anytime soon anyway. Global warming is a much more immeinent risk. Also, if it were free-falling around the Earth, then you would have to know all the GRBs years in advance so that the disk could be positioned correctly. In reality, we probably wouldn't know about a GRB until the instant it hits us, because in order to detect it, we would need access to the radiation, unless you set up probes around the solar system, which might give us just a few hours' warning (or not, because for the time the signals from the probes would have gotten to us, the probe would have been destroyed and the GRB would ahve already hit us). This again would take decades and trillions of dollars. It's also pretty hard to tell when a particular part of the sky or star is about to go GRB, as when we see the star exploding, if it is releasing a GRB, it would have already hit us by the time we detect the light from the supernova. Also, if that disk hits Earth, its impact would be spread across the Earth, but can you imagine a metre-thick material crashing towards Earth at 15 km per second (9 miles per second) on every part of the half of the Earth? Sure, parts of it might burn up, but large fragments would still land all over the Earth, and there's another trillion dollars wasted. Besides, if an asteroid hits that disk? It's going to break if it's only a few meters' thick. Also, an average single gamma ray will be blocked by 10 km roughly of air, but with relativity and zillions of rays, anything can happen. It usually isn't a problem unless the ray is directly pointed at us or if a large supernova explodes very nearby. Hope this helps. Thanks. ~AH1(TCU) 15:44, 9 March 2008 (UTC)[reply]
"Now, an object that big would weigh roughly 5x1014 tons if I calculated correctly," I don't think you calculated correctly. A cylinder 6378 km in radius, 1 m high, and with a mass of 5x1014 tons would have a density of only 3.9 g/cm^3. To block gamma radiation, the radiation needs to hit the atoms in the nuclei. Therefore, the nuclei have to be as large and heavy as possible. Lead is often used as a radiation shield for x-rays and gamma rays; its density is 11 g/cm^3, so the weight of the disk would then be 1.4x1015 tons. U-238 is much better, with a density of 19.1 g/cm^3; the disk would then be 2.4x1015 tons. --Bowlhover 04:38, 11 March 2008 (UTC)
To get to the end part of the question, no, there are no serious discussions in the scientific community to protect Earth from a GRB. As noted by the responses above, it's impossible short of some sort of Dyson sphere. However, there is one agreed-upon method to protect humanity from a GRB -- extrasolar colonization. — Lomn 17:45, 9 March 2008 (UTC)[reply]
Wouldn't a more realistic way to protect humanity be for us to live underground ? This would also have the advantage of providing protection from the far more likely scenario of a nuclear war. Environmentalists would be happy as underground living means less energy used for heating and A/C and the surface could be reclaimed and used exclusively for farms and nature. StuRat (talk) 17:56, 9 March 2008 (UTC)[reply]
Unless you can move the entire ecology (or at least a fully self-sustaining portion thereof) underground, the GRB is still going to fry life on the surface. — Lomn 22:07, 9 March 2008 (UTC)[reply]
Nuclear reactors could provide power almost indefinitely. Greenhouses could maintain plant life. Animals could be bred and slaughtered... -- 81.98.253.215 (talk) 22:35, 9 March 2008 (UTC)[reply]
Aren't you concerened about a mine shaft gap, though?
Atlant (talk) 16:36, 10 March 2008 (UTC)[reply]
Hi. How would extrasolar colonization help protect against GRBs? They are probably just as likely to hit that extrasolar planet as on Earth. Also if we were to create a dyson sphere then we would have become so technologicly advanced that we would have destroyed our own ecosystem and then we would have to colonise other planets. Thanks. ~AH1(TCU) 23:45, 9 March 2008 (UTC)[reply]
It's simply a matter of not having all one's eggs in one basket. You're entirely correct that any planet is vulnerable. — Lomn 18:24, 10 March 2008 (UTC)[reply]

How long does a GRB last? If less than a few hours, I'd submit that about half the Earth will be naturally shielded by miles of earth/Earth.

Atlant (talk) 16:37, 10 March 2008 (UTC)[reply]

As I understand it, it's not the immediate deaths from irradiation that make it an extinction threat, but the atmospheric changes it would cause. But there's not likely to be one close enough to us to matter anyway; as I recall it would have to be within about 500 light-years, and there just aren't any candidates within that sphere. There's more risk, I'd say, that we'll destroy our ozone layer ourselves. Minor planets, on the other hand, remain a genuine threat. --Trovatore (talk) 19:00, 10 March 2008 (UTC)[reply]
A gamma ray burst lasts seconds, not hours, and from the sources I have seen, a direct hit from a burst 8,000 light years away could cause changes in the atmosphere, but this is obviously theoretical. Jehochman Talk 19:19, 10 March 2008 (UTC)[reply]

Another birdsong?

Hi. This morning at around 8am (because of DST), I heard another birdsong. This time it was about -12C outside at sunrise, after a major snowstorm that dumped 25cm of snow and created 3 ft drifts. Anyway, it sounded like it was coming from a tree. First there were either one or two or three whistle-like notes, then six or seven flatter notes that sounded like a mix between a chirping bird's chirp, a pigeon's coot, and an owl's hoot. So for example: Whree? Whree? woo-woo-woo-woo-woo-woo-woo. The entire call lasted just over three seconds. It is sometimes repeated once every ten seconds maybe. I don't think it's an owl, and aren't owls nocturnal anyway? Any idea what it might be? I live in southern Ontario. Thanks. ~AH1(TCU) 14:27, 9 March 2008 (UTC)[reply]

Your description doesn't match exactly, but listen to the call of the mourning dove here and let us know if that's it. Most owls are nocturnal, but not all. --Milkbreath (talk) 14:47, 9 March 2008 (UTC)[reply]
Hi. No, that probably isn't it. The one I heard had more ooos and sounded less mournful. The first note I heard did not rise up so suddenly. The ooos I heard did not lower in pitch as they were sung, they stayed constant. Also, the one I heard was a bit higher in pitch, and there was often more than one starting note. By the way, I think I have heard that mourning dove song somewhere, and now I know why they call it the mourning dove. Thanks. ~AH1(TCU) 15:27, 9 March 2008 (UTC)[reply]
I don't think I know from personal knowledge what bird you've got there, but I'm trying to find it. The call of the saw-whet owl comes close. The northern pygmy-owl is diurnal and has a call like yours, except in the reverse order. A nocturnal bird will call in the daytime sometimes. --Milkbreath (talk) 16:19, 9 March 2008 (UTC)[reply]
Hi. Yesterday when I was outside, I heard about two dozen different bird calls. I actually heard an owl's call, and no this one did not sound like an owl. In fact, the whole thing resembled a chirp. I noticed the "woo" part actually sounded more like a "doo" or a "dchrew". Sometimes, there were as few as one or as many as eleven of these. I also saw some crows chasing a larger bird. There were many exotic calls, and I'm not going to ask for all of them. Ones like, "tchit-thcit-tsi-TCHEE!!!" and RDEW! RDEW! rd-RDEW!", "eeee! eeee! eee-ee!", a descending "oh-ree oh-ree oh-ree oh-ree oh-ree", a high-pitched squeal, and one that sounded like a shrill cry. I even heard one that sounded like a rattlous chirp, one that sounded like a baby crying, and one that sounded like wings flapping and chirpping at the same time for a dozen notes per half a second. I identified calls of crows, owls, seagulls, and geese. Possible sourses for some of the calls may or may not include: Red-Tailed Hawk, Northern Harrier, American Kestrel, Common Snipe, Greater Yellowlegs, Least Sandpiper, Herring Gull, Ring-Billed Gull, Bonaparte's Gull, Mourning Dove, Rock Dove, Eastern Screech-Owl, Long-Eared Owl, Great Horned Owl, Boreal Owl, Belted Kingfisher, Northern "Yellow-Shafted" Flicker, Red-Bellied Woodpecker, Downy Woodpecker, Hairy Woodpecker, Horned Lark, American Pipit, Black-Capped Chickadee, Red-Breasted Nuthatch, Brown Creeper, Winter Wren, Carolina Wren, Ruby-Throated Kinglet, Eastern Bluebird, American Robin, Northern Mockingbird, Blue Jay, American Crow, Northern Shrike, Cedar Waxwing, Yellow-Rumped Warbler, Northern Cardinal, Red Crossbill, White-Winged Crossbill, Eastern Towhee, Evening Grosbeak, Pine Siskin, Common Redpoll, House Finch, Purple Finch, Pine Grosbeak, White-Throated Sparrow, Field Sparrow, Swamp Sparrow, American Tree Sparrow, Fox Sparow, Song Sparrow, Dark-Eyed Junco, Snow Bunting, Lapland Longspur, Red-Winged Blackbird, European Starling, Eastern Meadowlark, Brown-Headed Cowbird, etc. Thanks. ~AH1(TCU) 15:07, 15 March 2008 (UTC)[reply]

Tortoise speeds

The common stereotype is that tortoises are slow animals. Form what I have observed from my german tortoise, this does not seem the case. For their size, ad the weight of their shell, the seem to have a relatively good speed. Does anyone have some outside input on this? —Preceding unsigned comment added by Gbgg89 (talkcontribs) 16:55, 9 March 2008 (UTC)[reply]

I would call tortoises slow, yes. Saying "for their size and weight of their shell they are fast" misses the point, in my opinion. That's like saying "may car is fast considering it's minuscule engine, poor gear ratios, and high mass". However, we can find other larger and armored animals which are much faster, like a rhino. I'd say the short legs are what slows tortoises down. Are tortoises fast enough to outrun any predators ? No, they must rely on their shell to protect them. Perhaps we should turn this around and conclude that they are slow because, due to their armor, they don't need to be fast to escape predators. StuRat (talk) 17:48, 9 March 2008 (UTC)[reply]
Ignoring WP:NOR, I would say the pet tortoise I have is not only slow because it cannot move quickly; it's also slow because it doesn't move much. It has an inactive life style and, except for eating, only very occasionally transports itself. --Bowlhover 04:52, 10 March 2008 (UTC)

How do astronauts swallow ?

Gravity seems to be an important factor in swallowing on Earth. I know I can't swallow when upside down and find it difficult when lying down. This is particularly true of difficult to swallow items like dry crackers. Related to this is the question of gastric reflux. This often happens to people while lying down, since they lose the gravity assist in keeping stomach acid in the stomach. Wouldn't this be even worse for astronauts, who always lack that assist while in orbit ? StuRat (talk) 18:08, 9 March 2008 (UTC)[reply]

Well, swallowing is largely due to peristalsis, which should be unaffected by orientation. In fact, in zero G, it ought to be easier to swallow than lying down or upside down in normal G, since you're not working against gravity, though harder than upright. -mattbuck (Talk) 18:25, 9 March 2008 (UTC)[reply]
That would explain how an astronaut's body would handle excretions too. Julia Rossi (talk) 21:35, 9 March 2008 (UTC)[reply]
You've brought this up before, and I'll say again that swallowing being completely independent of your orientation is one of the very first experiments I remember encountering in a children's science book. Gravity is not supposed to be a factor in anyone's swallowing. Skittle (talk) 19:15, 12 March 2008 (UTC)[reply]
StuRat, you are correct and the others are not quite right. Gravity matters--to a point. If you think about it from an evolutionary standpoint, it makes sense. Since we swallow with a gravity assist, there is no selection pressure for the esophagus to work in non-gravity or anti-gravity environment. So even when you are lying down, the esophagus has to work harder to get the bolus down by producing multiple waves of peristalsis (secondary peristalsis). Still, even with gravity, swallowing would be hard without a working esophagus. This happens in a condition called achalasia. On the issue of reflux, it actually results from inappropriate relaxations of the lower esophageal sphincter. Mauricev (talk) 09:42, 8 August 2009 (UTC)[reply]

Is it possible for someone of mixed race to be born transparent as a result? Why not?

Assuming the colors of his or her parents blend perfeclty, would it be possible to pick two such people that their son or daughter would (under the condition just specified) be born transparent as a result? Why or why not? I've tried to read our color article but it's a bit hard to take in all at once... —Preceding unsigned comment added by 79.122.37.156 (talk) 18:27, 9 March 2008 (UTC)[reply]

Not due to blood in the skin it would seem. Though this is based on a very rudimentary understanding of matters. Our article skin tone ny156uk (talk) 18:39, 9 March 2008 (UTC)[reply]
No. Skin cells are not transparent and in fact between people of different races they are almost identical with the main cause for difference in skin colour between these races being the saturation of melanin in ones skin, something dark skinned people have more of than white skinned people. Blood and other factors will always add colour to skin which is independent of race.PiTalk - Contribs 19:47, 9 March 2008 (UTC)[reply]
1. Colors of parents don't blend perfectly
2. Lighter skin is just an absence of melanin. There isn't a "transparent" option in skin, just a question of how much melanin. Total absence of melanin (see albinism) isn't transparent, it's just very very light.
3. You seem to be confusing additive color and subtractive color. Light is additive—it adds up to white; physical inks and dyes and melanin is substractive—it adds up to black. You can't mix white in a subtractive color system; white is the absence of color. --98.217.18.109 (talk) 19:46, 9 March 2008 (UTC)[reply]

When you look at a person's skin, the color you see is that person's phenotype. It's primarily determined genetically, but it's determined by a large number of genes (certainly more than six) which interact in complex ways. You can't determine exactly which genes (or alleles) a person has by looking at their skin. A child's skin color is determined by which alleles he inherits, and not by the skin color of his parents. It's not a matter of "blending" phenotypes, but of which alleles he inherits (at random) from his parents. There's no guarantee that the child's genes are those which have manifested in his parents. And there's no combination of genes that results in "transparency" in humans. - Nunh-huh 01:27, 10 March 2008 (UTC)[reply]

Of course some people claim to be transparent. With regard to skin, some animals are transparent, though I don't think we have an article about this.--Shantavira|feed me 10:39, 10 March 2008 (UTC)[reply]
Tansparent animals are usually those that live in water. For things that live in air there is a big refractive index change from air to the flesh of the animal. Not only that but the skin on the surface is not completely smooth, and has air surrounding the dead cells. This results in light being scattered, and gives a white appearance. So if there is no colour in the skin, it will appear white. Light penetrating further in can be scattered by small organells etc, so that can give a bluish appearance. A human is so thick and full of light scattering, and coloured objects there is no way they will be transparent. Even the transparent animals are very small. Graeme Bartlett (talk) 22:36, 10 March 2008 (UTC)[reply]

Thermosynthesis & Radiosynthesis --

Definitions -

By these terms, I refer to processes similar to photosynthesis, but harnessing thermal or other radiative energy in the place of visible light.

Comment -

The natural origin of sucrose is living-plant photosynthesis, energized by visible light -- whereby (typically) sunlight, water and carbon dioxide (from the air) are reacted. I suspect that sucrose is an overall very efficient energy storage medium which is easily transported.

Question -

Does anyone know of any other radiant-energy process other than visible-light photosynthesis which can produce sucrose or some other simple, easy-to-handle hydrocarbon ? I suspect that such processes could possibly provide energy storage, transport and recovery mechanisms far superior to those based on hydrogen technology. 208.63.237.3 (talk) 18:45, 9 March 2008 (UTC) allenwoll[reply]

There are organisms that use chemosynthesis, but it is similar to photosynthesis as far as glucose creation is concerned. For storage, plants make polysaccharides, such as starch and pectin. Animals make glycogen and fat (a hydrocarbon). I'm not sure about your claim to a far superior mechanism, because burning oil is about as efficient as you can get using biological materials (as far as I know). The only problem is that this releases a lot of CO2. Hydrogen fuel is an entirely different and efficient (albeit, slightly dangerous) way of storing energy. Generally speaking, if you're looking for high efficiency for storing and using energy, do not look at biological systems. They're good, but we already know of better. (EhJJ)TALK 23:42, 9 March 2008 (UTC)[reply]
+++++++++++++++++++++++++++++++++++++++++++++
Regarding CO2 -- That is not an issue, since the manufacture of the sucrose used CO2 from the ait in the first place : no net change.
Regarding Storage & Transport -- A bag of sucrose looks pretty transportable and storable to me !! Starch is OK, too, but what would be the advantage ??
Regarding Biological SYSTEMS -- I wasn't considering LIVING systems, rather processes like what has become generally known as artificial photosynthesis.
Regarding Hydrogen -- The hydrogen storage/transport cycle is VERY inefficient at this time, as well as dangerous. So was that for acetylene at one time, but no more. But we are not there yet for hydrogen and may never get there, TBD.
"Know of better " -- What, then ??
+++++++++++++++++++++++++++++++++++++

. —Preceding unsigned comment added by 208.63.237.3 (talk) 00:03, 10 March 2008 (UTC)[reply]

Sucrose is not a hydrocarbon, but a carbohydrate (and the former would yield more energy per mass and per volume if burnt with oxygen, thus they are more efficient from a storage point of view; biological organisms also use fat, which are more efficient than carbohydrates but less efficient than hydrocarbons in this respect). As for other carbohydrate-producing processes, some fungi are suspected to use gamma radiation. Icek (talk) 03:51, 10 March 2008 (UTC)[reply]

++++++++++++++++++++++++++++++++++++++++

Thanks, Icek.
Let me rephrase -- I seek information on the potential existence of a process using thermal or other radiation to produce "optimum" chemical compounds from readily available, cheap and nominally non-toxic materials -- water, air (including carbon dioxide), etc -- "optimum" for energy storage and transport as well as for energy recovery. Such a process may or may not entail living organisms, but perhaps better not.
Whether the compound is a hydrocarbon, a carbohydrate or a fat -- or some other sort of compound altogether -- is, I think, irrelevant, except as to its potential performance toward the stated goal.

++++++++++++++++++++++++++++++++++++++++ . —Preceding unsigned comment added by 208.63.237.3 (talk) 05:09, 10 March 2008 (UTC)[reply]

Unfortunately, our article on thermosynthesis is just a single line. That said, there are some interesting hits if you Google search "thermosynthesis" [4], or take a look at PubMed. Unfortunately, I can't help you find industrial processes used for converting thermal energy directly in to chemical energy, as I only know of methods that involve an electrical energy intermediary step. Good luck! (EhJJ)TALK 13:39, 10 March 2008 (UTC)[reply]

Query: Name of inedible string/fiber found in bananas?

I am a writer looking for the name of the inedible string/fiber that runs along the length of a banana (often left after the peel is removed).

I have been unable to find it anywhere.

Thanks in advance

Writerdog8 (talk) 20:34, 9 March 2008 (UTC)[reply]

I don't think any part of a banana is inedible? --BozMo talk 20:39, 9 March 2008 (UTC)[reply]
It seems to be part of the "pith" or inner rind inside the skin that takes a second stripping if you don't like it on the fruit. Julia Rossi (talk) 21:25, 9 March 2008 (UTC)[reply]
Its going to be equivalent to a leaf vein, or Vascular tissue, ie xylem and phloem. Graeme Bartlett (talk) 22:40, 10 March 2008 (UTC)[reply]

What plant are these people selling?

http://pics.livejournal.com/n0mad0/pic/0000gk7z --Sonjaaa (talk) 20:45, 9 March 2008 (UTC)[reply]

Russian "kale", perhaps? As here [5]. Julia Rossi (talk) 21:18, 9 March 2008 (UTC)[reply]

Lithium sunsets?

A co-workers claims to know why sunsets are pleasing to the eye. He claims that sunlight, when traveling through the large amounts of atmosphere (due to the sun's low angle relative to the horizon) somehow contains micro amounts of lithium that enter the eyes and makes the sunset watcher feel good. That sounds absolutely preposterous. Wouldn't landscapes then be encrusted with elemental lithium from the countless centuries of sunsets? Wouldn't particles of lithium be blocked by contact lenses, glasses, and sunglasses -- so only bare eyes could enjoy sunsets? --70.167.58.6 (talk) 22:26, 9 March 2008 (UTC)[reply]

You don't need to come with reasons why his idea is stupid. He needs to come up with ideas of why his idea isn't. Theresa Knott | The otter sank 22:28, 9 March 2008 (UTC)[reply]
Wiki doesn't have an article on everything, contrary to WP:WHAAOE: there's no article for chain yanking!
Atlant (talk) 16:40, 10 March 2008 (UTC)[reply]
More concerningly - we have no article on floor buffers (or at least that I could find)! --Kurt Shaped Box (talk) 00:17, 12 March 2008 (UTC)[reply]

How can sunlight contain lithium? Sunlight are photons and lithium are atoms. 122.107.151.153 (talk) 23:35, 9 March 2008 (UTC)[reply]

There is water on the exterior of the eyes, and when lithium comes into contact with it, it forms lithium hydroxide, hydrogen, and energy in the form of heat (an exothermic reaction). I don't know how lithium in the eyes would feel, and nor do I want to. Lithium hydroxide is highly caustic and the temperatures are hot enough to burn.
There won't be buildup of lithium on the ground, though, because any contact with water will cause it to immediately become lithium hydroxide and hydrogen. The hydrogen would escape to the very outer limits of the atmosphere, but does anyone know what will happen to the LiOH?
As for reasons why your friend's theory is ridiculous:
(1) Lithium cannot pass through the atmosphere without reacting with the oxygen and forming lithium oxide.
(2) How can the lithium pass through the atmosphere almost horizontally without falling due to gravity or stopping due to air resistance? What is propelling them?
(3) Why is it that the lithium only exists when the Sun is low on the horizon, and not when it's high in the sky? Ask your friend to explain this.
(4) Sunsets do not look any different through optical aid (your reason).
(5) Your friend was probably ascertaining your guillibility. --Bowlhover 01:04, 10 March 2008 (UTC)
According to him, the sunlight doesn't contain lithium, but carries/propels the lithium that's formed by some reaction between sunlight and low angle atmosphere. I guess these particles are being wisked into my eye carried on beams of sunlight. SO... just why are sunsets pleasant to look at? --70.167.58.6 (talk) 03:41, 10 March 2008 (UTC)[reply]
I'm guessing because they're beautiful. 206.252.74.48 (talk) 12:51, 10 March 2008 (UTC)[reply]
There has to be some kind of psycho-social theory regarding why so many distinct cultures all find a sunset aesthetically pleasing. It seems counter to the general notion of sun worship, as the sun is leaving for a while. This topic could probably result in several academic publications, if I were a student in the humanities - "Sociocomparative aspects of the aesthetic appeal of solar activity," "Anticipation of diurnal solar rearrival as a cultural indicator of Freudian delayed pleasure-principle," etc. Nimur (talk) 16:09, 11 March 2008 (UTC)[reply]
Cute titles but those read like science or social science papers. Real humanities papers are of the format "Something witty (bonus points if you make it a variation of "A Tale of Two Cities" or some other bad Dickensian pun): Something more specific, but not scientific sounding." --98.217.18.109 (talk) 23:57, 11 March 2008 (UTC)[reply]
Does your friend have any alternative medicine products based upon his theories in the pipeline, as a matter of interest? --Kurt Shaped Box (talk) 00:15, 12 March 2008 (UTC)[reply]
I wouldn't want to see him down his elemental lithium tablets with a glass of water. bibliomaniac15 00:20, 12 March 2008 (UTC)[reply]
Oh, but I would. It's surprisingly how exothermic the reaction is and how caustic LiOH is, especially when both are sensed with the tongue. --Bowlhover 15:34, 12 March 2008 (UTC)
        My understanding is that sunlight does not contain lithium, but exposure to natural sun effects        your circadian/biological clock, and therefor mood.  We know lithium is found in the body, and is effective at treating bipolar disorders.  Atmosphere has nothing to do with it.  More info here; http://www.psycheducation.org/mechanism/Clock.htm

This week's meteor

Hi. I saw on the news that an astronomer reported that a meteor this week may have landed in Georgian Bay near Parry Sound, Ontario. Do we have either an article, a mention in an existing article, or a Wikinews article on this? Should this be mentioned somewhere perhaps in the meteorite article? Thanks. ~AH1(TCU) 23:36, 9 March 2008 (UTC)[reply]

Well, I suppose it might warrant a mention, but meteorite impacts aren't really that rare, it's just we don't tend to notice them. -mattbuck (Talk) 01:06, 10 March 2008 (UTC)[reply]
I don't think this meteor is notable enough to be mentioned in meteorite. There is nothing extremely unique about this specific rock; it hasn't even been recovered yet. Many much more significant meteorites exist, such as ALH84001, suspected of containing Martian life, and the Kaidun meteorite, which could have come from Mars' moon Phobos. --Bowlhover 05:28, 10 March 2008 (UTC)

Hawking radiation emission rate inversely proportional to black hole mass - why?

The rate of Hawking radiation emission increases as the mass of the black hole decreases([[6]], second paragraph), what I don't understand is that if the emission method ([[7]]) is related to zero-point energy particles appearing and one half of the pair being captured, why is it that when the surface area of the event horizon would be greater for a larger black hole the Hawking radiation doesn't increase? To my lay understanding the probability of the particle capture would increase. 86.5.95.29 (talk) 23:51, 9 March 2008 (UTC)[reply]

Well, regardless of black hole size the ambient temperature goes to infinity as you approach the event horizon. The usual Hawking temperature is the limit at large distances. So I guess you could say that a larger hole emits less radiation because more of it gets pulled back in, although I don't know if that's a correct picture. -- BenRG (talk) 00:35, 10 March 2008 (UTC)[reply]


March 10

Circular Motion

Imagine that you swing about your head a ball attached to the end of a string. The ball moves at a constant velocity in a horizontal circle. Can the string be exactly horizontal? Why? - Thanks —Preceding unsigned 00:23, 10 March 2008 (UTC)

Is this homework? Try setting up the problem with a horizontal string and working out all the forces involved. (There aren't too many—gravity, tension, centripetal/centrifugal.) -- BenRG (talk) 00:38, 10 March 2008 (UTC)[reply]

The answer is "no", and the presence of a downward gravitational force vector should make the reason why obvious. =Axlq 01:55, 10 March 2008 (UTC)[reply]

Sure it can. The swinger just has to provide enough force to counteract gravity. Oops. Clarityfiend (talk) 02:25, 10 March 2008 (UTC)[reply]
No it can't as would be obvious if you draw and diagram and add the forces involved. Theresa Knott | The otter sank 05:21, 10 March 2008 (UTC)[reply]
Nitpick alert: the ball can't be moving at constant velocity either, though it may be moving at constant speed. Constant velocity requires travel in a straight line. AlmostReadytoFly (talk) 09:09, 10 March 2008 (UTC)[reply]

Energy Drink Death

First off, I know that drinking enough of anything can kill you.

My question is though, our article on Cocaine (drink) states that it contains 350% as much caffiene as redbull. That seems like a lot to me. How much caffeine can the human body tolerate? (I am very disinterested in killing myself, this is just curiosity) —Preceding unsigned comment added by 24.128.192.184 (talk) 00:35, 10 March 2008 (UTC)[reply]

Caffeine. Quote: "The LD50 of caffeine in humans is dependent on weight and individual sensitivity and estimated to be about 150 to 200 milligrams per kilogram of body mass, roughly 80 to 100 cups of coffee for an average adult taken within a limited timeframe that is dependent on half-life" Wisdom89 (T / C) 00:51, 10 March 2008 (UTC)[reply]

Wouldn't the LDLo or LDmin be more appropriate than the LD50 in this case? —Keenan Pepper 20:15, 10 March 2008 (UTC)[reply]
The drink seems to have 280 mg of caffeine per serving. So that means you'd start getting into the dangerous range after 15 consecutive (and immediate) servings or so (when you start getting around 5 g of caffeine), depending on your weight. Which seems like a lot, but when compared to coffee, that's about an order of magnitude more toxic. --98.217.18.109 (talk) 15:21, 12 March 2008 (UTC)[reply]
Not quite an order of magnitude, more like less than twice (but yes, that's still quite a bit). Coffee contains an average of about 135mg of caffeine in 240mL, while Cocaine has 280mg per 240 mL (Red Bull has 80mg in the same volume). – ClockworkSoul 16:04, 12 March 2008 (UTC)[reply]
Note that most energy drinks in NZ recommend a 2 cans max daily intake. However the reason according to a number of sources (I believe one can said it and this source [8] also supports it) is not the caffeine level but the Vitamin B12 level. However Vitamin B12 doesn't mention anything about potentially lethal effects. But most energy drinks here have a level of caffein similar to a cup of coffee anyway (according to the can)... Nil Einne (talk) 16:49, 15 March 2008 (UTC)[reply]

Hair loss (non-balding)

Hi. This is not a request for medical advice. I think I've heard somewhere that the average adolescent loses roughly 120 hairs a day. Is this true? We really should have an article on hair loss that does not redirect to balness, then. Is that why when I drop food on the carpet it gets covered with hair even if it has been vacuumed? When does hair loss of more than one hair a day usually begin at, then? On average, when is the first white, gray, or silver hair occur in an average person (head)? Do people lose skin hairs too (not caused by accidental pulling)? This is not homework. On average, approximately how many hairs would have been lost (on a person that is not born will to be bald) and how many white/gray/silver hairs would have developed by the time an average person turns 18, for example? Do common mammalian pets (eg. cats, dogs, rabbits, hamsters, gerbils, mice, rats, etc) lose hair as rapidly as humans or more rapidly? What colour of hair loses hair most rapidly and which least rapidly? What about de-melanined hairs? Thanks. ~AH1(TCU) 00:49, 10 March 2008 (UTC)[reply]

These links should help you with your questions: [9],[10], [11]. Wisdom89 (T / C) 00:54, 10 March 2008 (UTC)[reply]
Anyone who has ever owned a cat will tell you how rapidly and in vast quantities they lose hair. As for the carpet, it's probably a mixture of hair which wasn't picked up by the vacuum, carpet fibres and other random dust and detritus. -mattbuck (Talk) 01:02, 10 March 2008 (UTC)[reply]

What is naphtha?

Is it really the same thing as petroleum ether? If so, why aren't the articles merged? If not, what is the difference? What is the main meaning of naphtha when the term is used in modern English? --Sonjaaa (talk) 00:55, 10 March 2008 (UTC)[reply]

Petroleum ether is obtained from petroleum refineries as the portion of the distillate which is intermediate between the lighter naphtha and the heavier kerosene says the PE article, while Naphtha says it is referred to as PE. My guess is that naphtha is not the same, but that there is a blurred boundary. -mattbuck (Talk) 01:00, 10 March 2008 (UTC)[reply]
All the terms used by the petro-chemical industry for the various fractions produced by oil refineries are ill-defined chemically and always include a large range of alkanes which overlap quite considerably with neighbouring fractions. It is more of a relative order of fractions up the distillation column which defines the terms. SpinningSpark 08:02, 10 March 2008 (UTC)[reply]

Question

Why do scientists use animals' Latin names? 58.168.209.250 (talk) 01:17, 10 March 2008 (UTC)[reply]

Biological_nomenclature#Value_of_binomial_nomenclature --Sonjaaa (talk) 01:20, 10 March 2008 (UTC)[reply]
That doesn't answer why Latin in particular, or the historical reasons for it. The simple answer is that Latin used to be the lingua franca of science—the "neutral" language that scientists of all countries used when they wanted to communicate with one another (though by the 19th century it was French and German; today it is English). It was expected that all educated men (and it was primarily men who were allowed to participate in the republic of letters for a good long time) would know the language, whereas whether they would know English, French, Russian, Italian, Swedish, etc. was less likely. Thus a large number of the early major scientific publications were originally written in Latin as well—De revolutionibus orbium coelestium, Philosophiæ Naturalis Principia Mathematica, etc.
The practice of using Latin specifically for binomial nomenclature (those animal names you are thinking of: genus plus species, Homo + sapiens) goes back to Linneaus, the father of modern taxonomy, and has stuck every since. Standardizing nomenclature in a neutral, non-changing language was important to making sure that a creature in one part of the world was really the same (or similar) creature in another, even though they had radically different names in the local languages. --98.217.18.109 (talk) 01:45, 10 March 2008 (UTC)[reply]
I've always enjoyed the fact that "Linnaeus" is itself a Latinized Swedish name. I suppose as "Carolus Linnaeus" it would be fully binomial. - Nunh-huh 01:42, 10 March 2008 (UTC)[reply]
Well, again, it's the language Linnaeus himself published his major works in. He wrote the whole thing in Latin, name included. He practiced what he preached! The custom of using only the Latinized last name for "major" writers is quite old (think also Copernicus). Scientists already drop the Albert from Einstein quite often; imagine how that'll progress in two hundred years. --98.217.18.109 (talk) 01:45, 10 March 2008 (UTC)[reply]
Yes, it's not that he was unusual in adopting a Latin name (Paracelsus, Helvetius, etc.) but that it was in his case an amusing coincidence. - Nunh-huh 02:05, 10 March 2008 (UTC)[reply]
There is a second reason Latin was used - it is a "dead" language. It is not changing due to modern usage. So, the definition of a Latin word 100 years ago is the same definition today and will be the same definition 100 years from now. English (or any other modern language) doesn't have that luxury. Consider "urchin". What is that? Easy, a nasty street kid. Or, perhaps it is one of those prickly things in the ocean. Not long ago, it was a prickly little animal in British gardens. Before that, it was simply a vulgarity. Who knows what it will mean in another 100 years. It is a good thing we don't use it for scientific purposes since it isn't nice enough to pick one definition and stick with it. -- kainaw 17:04, 10 March 2008 (UTC)[reply]
Latin is mostly dead. But it is the official language of the Vatican, and that state's panel of Latin experts do formulate new words and constructions for things like "helicopter" and "hard disk" that the Romans knew not of. There are (or was) also available on CD the Elvis Presley classics, "Tenere me ama" ("Love Me Tender") and "Nunc hic aut humquaum" ("It's Now or Never"). - Nunh-huh 23:47, 10 March 2008 (UTC)[reply]
One big advantage of standarized bionomials: it's great for organization without the level of ambiguity and repetition of common names. Common names are often misleading (flying fox), used for a wide variety of different species (razor clam), or there may be several different names for a singe animal (cougar/puma/mountain lion/panther). A single, agreed-upon term helps everyone know just what animal we're talking about. — Scientizzle 20:19, 12 March 2008 (UTC)[reply]

Many people claim that houseplants "cleanse the air", by removing particulates and carbon dioxide and adding oxygen. While this is certainly true to some degree, I'm of the opinion that the quantity of plants needed to do a significant amount of this would be far more than would fit in the home, based on plants having much slower metabolisms than people. So, I'd expect that if the biomass of people and pets in your home is 1000 lb, that you would need maybe 10,000 or 100,000 lb of plants to clean the air of the carbon dioxide and other waste products the people create. How can I calculate the actual ratio ? StuRat (talk) 01:24, 10 March 2008 (UTC)[reply]

Well, this would take some research - if you can come up with the amount of CO2 used per day by plants, we could do a quick and dirty calculation. On the human side: average CO2 content in inhaled air is roughly 383 ppm by volume, which is 0.0383%.
Average CO2 content in exhaled air is 4.5% by volume. The difference (4.5% minus 0.0383%) would be 4.1%. So the CO2 output from a human is 4.1% of the volume of inhaled (or exhaled) air.
The tidal volume (volume of a normal breath) of a human is about 500 mL. The usual respiratory rate varies between 12 to 20 breaths per minute. For our purposes, let's say 18 breaths per minute.
There are 1440 minutes in a day, or 25920 breaths per day. 25920 breaths of 500 mL each = 12960000 mL = 12960 liters per day. 4.1% of 12960 liters is 531 liters of CO2 produced per human per day. There are 22.4 liters/mole of gas at STP, so 531 liters is 23.7 moles. There are 44 g/mole of CO2, so 23.7 moles = 1043 grams of CO2, or roughly one kilogram, produced per human per day.
I think :) - Nunh-huh 02:02, 10 March 2008 (UTC)[reply]
4.5% minus 0.0383% is 4.46%, not 4.1%. Oh, how you curse us, Mr. Decimal Point.-RunningOnBrains 21:13, 12 March 2008 (UTC)[reply]
Good work, now we need some info on the plant side. StuRat (talk) 03:15, 10 March 2008 (UTC)[reply]
You also need to account for the efficiency of the plant at converting CO2 to oxygen. I have read that plants are extremely efficient (95%[12]) at using light energy for this process. So even if plant metabolism is slower, it may be compensated by a more efficient CO2 to O2 conversion than a human's ability to produce CO2 from O2.
The amount of oxygen produced also depends on the light available. I had an aquarium containing hydrilla plants. When the sun shone on them, streams of oxygen bubbles would spew forth from the leaves. =Axlq 02:09, 10 March 2008 (UTC)[reply]
23.7 moles of carbon (C) is 284g, so the total carbon content of plant mass in your house must increase by 284g per day per human. If we assume that plants are reprented by the formula CH2O (sugars) it is 948g per person per day, but there are also phosphates and nitrogen that the plant takes in, so 1kg per day would be minimum...Which looks very unrealistic.--Shniken1 (talk) 02:31, 10 March 2008 (UTC)[reply]
Interesting approach. Aren't plants something like 95% water ? I wonder what percentage of their weight is carbon. If we figure 4% by weight, then one kilogram of carbon increase would mean 25 kilograms of plant mass increase. StuRat (talk) 03:15, 10 March 2008 (UTC)[reply]
Yeah water would make up significant amount of the mass of the plant so yeah the mass of the plant as a whole would have to increase a lot more than its carbon mass (depending on the plant), and as plants get bigger I would suppose that water would be harder to obtain (via transirpation) so it may be the limiting growth factor.--Shniken1 (talk) 03:52, 10 March 2008 (UTC)[reply]
I'd expect most of the water to be supplied by humans watering the plant, with the roots sucking it up from there. StuRat (talk) 23:55, 10 March 2008 (UTC)[reply]

Much of this depends on what you mean by cleanse the air. I would expect the impact on oxygen and carbon dioxide to be minimal unless you live in an air tight box. Transpiration is a likely reason for cleansing. Transpiration cools the air due to evaporation. It also adds moisture to the air. Both these could be beneficial inside a house. David D. (Talk) 18:55, 10 March 2008 (UTC)[reply]


Incidentally, the following link is in the "Biosphere 1, 2, 3, and J" section of our Biosphere article:

"Biosphere 3 (aka BIOS-3) - Experiment to be conducted starting Jan 2008 in the Gary C. Comer Geochemistry building at Columbia University's Lamont Campus in Palisades, New York."

However, when I follow the link, the article says:

"BIOS-3 was a closed ecosystem at the Institute of Biophysics in Krasnoyarsk, Siberia, in what was then the Soviet Union."

Which is correct ? StuRat (talk) 03:33, 10 March 2008 (UTC)[reply]

(after edit conflict) According to http://www.newton.dep.anl.gov/newton/askasci/1993/biology/bio027.htm, it takes around 353 plants to match the oxygen input rate that humans require. However, I have two concerns with the webpage:
(1) The webpage assumes that all oxygen breathed in that does not enter the inactive part of the lung (the "dead space") is absorbed. According to Breath#Composition_of_air, 15-18% of exhaled air is composed of oxygen by volume. For the 252 L/h of inhaled air that does not enter the dead space, and the corresponding 252 L/h that are exhaled, about 42 L is exhaled. Since 53 L/h of oxygen that does not enter the dead space is inhaled, this gives an hourly oxygen usage rate of 53-42=11 L/h. Assuming the plant calculations are correct, 76 plants would be needed.
(2) At the end of the webpage, the author states that "oxygen production decreases as carbon dioxide concentration increases". However, according to the photosynthesis article, the amount of CO2 plants take in is equal to the amount of O2 they output. Humans also necessarily expel as much CO2 as they breathe in oxygen, so if 76 plants provide enough oxygen to sustain a human, they also absorb enough carbon dioxide to keep the environment's CO2 level constant.
So 76 plants per person, and if we assume plants are 500 g on average, that's 38 kg/person. The plants would have to be properly maintained, of course, and cannot be allowed to grow; growth would mean a higher demand for CO2 than the humans can supply.
About BIOS-3: Most sources state that it was an experiment in the Soviet Union: [13][14][15]. Searching for "bios 3 'Gary C. Comer'" on Google yields only the Wikipedia biosphere article and mirrors of it. --Bowlhover 04:06, 10 March 2008 (UTC)
That seems like way too much oxygen production per plant kg. After all, a person weighs around 38 kg, and I have a hard time accepting that plants, with their far slower metabolisms, use CO2 at the same rate people use oxygen. Also, if the plant doesn't grow, where exactly does all the carbon go that it sucks out of the air ? StuRat (talk) 00:03, 11 March 2008 (UTC)[reply]
While it might matter for a biosphere, for your purposes as I understand them (the plant cleans up after the humans), it doesn't matter whether or not the plants grow. - Nunh-huh 00:44, 11 March 2008 (UTC)[reply]
But where does the absorbed carbon go if not to plant growth ? I suppose a fruit tree puts it into fruit and a nut tree into nuts (although this is a type of growth), but what about plants that don't produce food ? StuRat (talk) 17:29, 11 March 2008 (UTC)[reply]
List of air-filtering soil and plants may be of interest too. —Pengo 09:30, 11 March 2008 (UTC)[reply]
According to http://www.bio.net/bionet/mm/plantbio/2000-October/024096.html, the average plant produces 15.2 mL of oxygen per dm^2 of leaf area per hour. The "leaves" mentioned in the previous link I gave would then be 10 dm^2 each! Anyways, the total leaf area needed to support one person would be 750 dm^2, about 2.7 m x 2.7 m. I wonder how much that would weigh.
Also, the plants will grow, but I think they have to be trimmed so they don't use up CO2 at an excessive rate. --Bowlhover 15:18, 12 March 2008 (UTC)
Yes, that sounds more reasonable. Perhaps the leaves alone might weigh 38 kg, but including the stalks, branches, roots, etc., would make it much more. StuRat (talk) 17:01, 13 March 2008 (UTC)[reply]

We have had a similar question before. What if the person consumed plant material each day in the form of carbohydrates to total a significant portion of his daily caloric needs? Could the carbon in the plant material (carbohydrates) equal the carbon in the exhaled carbon dioxide the person emits? If the person exhaled more carbon than he consumed, wouldn't he become carbon deficient over time? Then there would be no net increase in the amount of carbon sequestered in the plant material. Ever see a demonstration of a sealed glass globe with water, plants, little fish and snails? The manage to keep recycling for quite a while sometimes until there is too much/too litttle sunlight or something else goes out of balance. Edison (talk) 14:04, 11 March 2008 (UTC)[reply]

I would expect that most of the carbon people eat ends up in their feces or urine. I'd expect animals would be needed to get this carbon back in circulation from there. Worms or insects can eat the feces, birds and frogs can eat them, and other animals can eat them, until we get to something people eat. StuRat (talk) 17:29, 11 March 2008 (UTC)[reply]
Some people might consume carbon in the forms of pencil graphite, diamonds, or charcoal and poop it out, but that is a small portion of the carbon consumed by humans.Human feces does contain a large proportion of undigested food which would still have much of its carbon content. Urine does incorporate some carbon from the carbon dioxide resulting from metabolism of food. Most carbon consumed is probably in the form of Carbohydrates, which are transformed along with some non-carbohydrates into Glucose by digestion, so that the cells of the body can use it as fuel. Glucose is (C6H12O6). It yields energy and exits the body as water and carbon dioxide. Long term space travel will likely involve a closed system of recycling the products of human and animal metabolism into new food such as the Biosphere 2 experiment tried to test. Edison (talk) 16:47, 12 March 2008 (UTC)[reply]

How can you ever fall in a black hole?

Since time slows per the reference frame of an outside observer as an object approaches the event horizon, and 'freezes' at the horizon (thus the Russian term for black hole: 'frozen star'), how can a black hole ever increase in mass? This was once considered a flaw in the concept of black holes, but evidently was somehow resolved. However, I've never seen an account of how it was resolved. (And yes, I do understand that for an observer falling past the event horizon, supposedly nothing unusual would appear to happen - except that you'd think s/he'd witness the end of the universe.) — kwami (talk) 07:03, 10 March 2008 (UTC)[reply]

This question has been previously answered here [16] and probably many times before. However, I never felt satisfied with the answer given then, for the same reason given by Kwamikagami. If someone can elaborate, please do. SpinningSpark 08:15, 10 March 2008 (UTC)[reply]
I’m no physicist. But as the mass of the black hole increases, the event horizon will increase in radius and objects that previously were hovering on the edge will be enveloped. And objects don’t need to actually be at the center of the singularity to contribute to the black hole’s mass/gravitational effect. (This is just my speculation.) — Knowledge Seeker 08:33, 10 March 2008 (UTC)[reply]
But the BH can't increase in mass unless something crosses the event horizon in finite time in the external reference frame. — kwami (talk) 08:46, 10 March 2008 (UTC)[reply]
I’m not sure I agree with that statement. — Knowledge Seeker 08:51, 10 March 2008 (UTC)[reply]
How would it increase in mass, without mass being added to it? — kwami (talk) 08:54, 10 March 2008 (UTC)[reply]
Perhaps I should be more clear. I contend that a mass placed just outside the event horizon will increase the gravitational field/curvature of space (as it would anywhere) and that therefore a small volume of space which previously had almost, but not quite enough gravity to have an escape velocity greater than c (or equivalent formulation) now will; in effect, the combined event horizon around the two objects will be larger. (Again, speculation.) — Knowledge Seeker 08:57, 10 March 2008 (UTC)[reply]

A group of massive objects, act as if all their gravity were concentrated at thier center of mass. So from a distant observer's POV anything frozen at the edge of an event horizon would still contribute to the total mass of the black hole system. Does that resolve the problem? Theresa Knott | The otter sank 12:28, 10 March 2008 (UTC)[reply]

Does this mean that an unfortunate astronaut just outside the event horizon might be observed by a "local" observer (say, another astronaut also falling into the black hole) as not having quite reached the horizon, but that both might be "observed" by a distant observer to be within the event horizon (i.e. part of the black hole)? dbfirs 12:54, 10 March 2008 (UTC)[reply]
Hi. On a Discover magazine there's this guy named João Magueijo that has a theory called Varying Speed of Light (VSL). When he introduced this theory, one physicist said it really stood for "Very SiLly". However, according to this theory, time and the speed of light slow near a black hole and never allow anything to escape nor go in (after all, black holes are infinitely dense). Hope this helps. Thanks. ~AH1(TCU) 14:22, 10 March 2008 (UTC)[reply]
As far as I know Magueijo's VSL theory is supposed to be an alternative to cosmic inflation, so it only matters in the very early universe and doesn't have any bearing on (present-era) black holes. -- BenRG (talk) 21:33, 10 March 2008 (UTC)[reply]

Objects can and do fall in. They only appear to stop at the event horizon. 64.236.121.129 (talk) 16:43, 10 March 2008 (UTC)[reply]

The objects may see themselves falling in. An outside observer sees the objects dim and disappear as they approach the Schwartzchild radius. An alternate theory Gravastar suggests that matter falling into a compact stellar object will appear to cool and form a Bose-Einstein condensate. One consequence of general relativity is that perception is affected by perspective. Jehochman Talk 16:51, 10 March 2008 (UTC)[reply]
64 and Jehochman, it’s not that Kwami’s really disputing whether they fall in or not (the phrasing of the heading, unfortunately, is misleading). His question is (as I understand it), essentially: If, from the reference frame of an outside observer, an object never passes the event horizon [due to time dilation], how/when will he observe an increase in the black hole’s mass?Knowledge Seeker 18:11, 10 March 2008 (UTC)[reply]
If matter has fallen inside the event horizon, how can you get back any information about mass inside? If light can't get out, neither can any other sort of information. If an outside observer can never see anything cross the event horizon, then presumably from their point of view, nothing ever has. A sphere or shell of mass around a point will be indistinguishable to an outside observer from all that mass being located at the center point. There would be no observable difference on the force of gravity affecting an outside observer. There are a lot of contradictions if we assume the existence of black holes. This may mean that our black hole model is wrong. Jehochman Talk 18:16, 10 March 2008 (UTC)[reply]

I don't know that much about black hole dynamics, but I think Knowledge Seeker is correct. Certainly it's a mistake to think that infalling matter has to fall through the event horizon to add to the mass, because nothing that happens inside the event horizon can be relevant to physics outside. Whatever it means for "the black hole to gain mass", it has to involve only the physics outside the event horizon.

The event horizon is acausal; it will happily expand faster than light to engulf infalling matter. Here's a special relativistic analogy. I'll use 2+1 dimensional Minkowski space because it's easier to visualize than 3+1 dimensions and 1+1 isn't enough for this example. At time t = 0 pick some region of the xy plane and magically destroy everything there. There's then a region of spacetime (a collection of events) with t < 0 which is unobservable to anyone who survives past t = 0, because any signal that might have originated from those events was destroyed. For example, say you destroy everything in the circular region x2 + y2 < 1 light year2. Then the unobservable region is a cone in Minkowski space whose apex is x = 0, y = 0, t = −1 year and whose base is the destroyed disc. If you destroy a square (|x| < 1 ly, |y| < 1 ly) then the unobservable region is a square pyramid. The boundary of the unobservable region is the event horizon. The event horizon is always a null surface because of how it was defined, but it may have "creases" which are not null. For example, in the square pyramid case you have corners which go from x = y = 0 at time t = −1 year to x = ±1 ly, y = ±1 ly at time t = 0; they're effectively moving at c√2.

Let S be the destroyed set. Then the observable region at time −t is , where Sc is the complement of S and Br(x) is the ball of radius r centered at x. Imagine S being eroded away from all sides at the speed of light as you go backward in time. Now say S is a sort of peanut shape, or an overlapping union of two circles. As you go back in time it will become a thinner peanut; then it will break into two teardrop shapes, with the pointy bits pointing toward one another; then the teardrops will shrink and become more circular until they disappear. Now run this forward in time: two expanding teardrops appear out of nowhere, extend toward each other faster than light, and merge. That's kind of what a black hole merger is like. The event horizon is not at all like a physical object obeying dynamical laws; it's a global property of the whole spacetime and it "knows about the future". In fact it's a theorem that the event horizons of two black holes that will eventually merge have the teardrop point on them from the beginning. That's for merging black holes, i.e. tossing one black hole into another, but I think tossing ordinary matter into a black hole works similarly. Whether any of this answers the original question I'm not entirely sure. This is a tricky subject. -- BenRG (talk) 21:33, 10 March 2008 (UTC)[reply]

I wonder if Knowledge Seeker might be on the right track: Once an object approaches a black hole close enough that their center of mass lies within the EH, does the EH then expand correspondingly? But this would happen while a small mass is still quite far from the EH, so it wouldn't be engulfed. (If you were to drop a sandwich into a Solar-mass black hole, it would hardly affect the EH at all, so we're left with my original question.)
It's not true that we can't know what's within the EH, only that no signal can be sent out. Fields extend beyond the EH, but any modulations of those fields would be red-shifted to a frequency of zero. However, with those fields we can still measure the total electronic charge, mass, and momentum (linear and angular). — kwami (talk) 22:23, 10 March 2008 (UTC)[reply]
Also, pace another comment, it isn't the case that the object just appears to slow down, due to some kind of optical illusion. It isn't light that slows down, but time itself. In the external reference frame, it really doesn't reach the EH. Thus the problem. — kwami (talk) 22:50, 10 March 2008 (UTC)[reply]

Intermolecular Forces

Can a molecule have hydrogen bonding, permanent dipole - permanent dipole and instantaneous dipole - induced dipole intermolecular forces acting upon it at any one time? 88.108.198.114 (talk) 17:38, 10 March 2008 (UTC) —Preceding unsigned comment added by 88.108.198.114 (talk) 17:37, 10 March 2008 (UTC)[reply]

Yes. something like liquid cholesterol would be a easy example to consider87.102.94.48 (talk) 17:45, 10 March 2008 (UTC)[reply]
True. Remember that molecules can be very very large (Protein), or even macroscopic (DNA), so different parts of one molecule can have completely different properties. --Bmk (talk) 17:59, 10 March 2008 (UTC)[reply]

Batteries in series vs. parallel arrangement

If one battery in a set is bad (low voltage), which arrangement is affected more? —Preceding unsigned comment added by 66.120.95.52 (talk) 19:22, 10 March 2008 (UTC)[reply]

Series. Voltages in series add, so a bad one will lower the voltage. In parallel, the remaining good batteries will try to supply whatever current it takes backwards through the bad battery to drop the supply voltage across it. If they can't supply enough current to do that, the voltage will fall, but if they can it won't, much. So a series arrangement is always immediately affected, and a parallel arrangement might not be affected very much at first. --Milkbreath (talk) 19:54, 10 March 2008 (UTC)[reply]
Could be either. If the one with low voltage is that way because the internal resistance has increased (low voltage under load, closer to normal voltage with no load, a common enough condition) then the parallel arrangement would work better, although the bad cell should be disconnected to avoid premature drainige of the others and because it is not contributing much to the output. If the bad cell has an internal short (happens sometimes) then it would drain the output of the others in parallel but would have less effect in series. In either arrangement the bad cell could explode. Edison (talk) 13:56, 11 March 2008 (UTC)[reply]
He asked about the case where one battery has low voltage, not where one battery is a dead short. "Could be either" is the answer to a different question, and is not a sensible answer to the question of "which" is affected "more". If it depended on the situation, "neither" or "it depends" would make sense. --Milkbreath (talk) 19:33, 11 March 2008 (UTC)[reply]

What bodily function is not possibel in space?

what bodily function is not possibel in space? —Preceding unsigned comment added by 89.243.182.3 (talk) 20:13, 10 March 2008 (UTC) - Please don't post in all caps, it comes off as yelling -- MacAddct  1984 (talk &#149; contribs)[reply]

pushing the Caps Lock key a second time? Thomprod (talk) 20:14, 10 March 2008 (UTC)[reply]
If you mean in a spaceship, all bodily functions are possible, but can get a bit messy. That's why they have special devices for going to the bathroom. -- MacAddct  1984 (talk &#149; contribs) 20:28, 10 March 2008 (UTC)[reply]
It hasn't been demonstrated that a child can grow in the womb in space. I'm not sure what people have thought about this though.Sancho 20:39, 10 March 2008 (UTC)[reply]
Larry Niven made a significant plot point out of this in his Known Space series. Pregnant women who lived in the Belt were required to live in a hollowed-out asteroid, called Confinement Asteroid, that was spun to provide centrifugal force in place of gravity. --Trovatore (talk) 20:53, 10 March 2008 (UTC)[reply]
Did they have another hollowed-out asteroid to send women with PMS ? :-) StuRat (talk) 17:35, 11 March 2008 (UTC)[reply]
Bodily functions remain intact, however, there can be serious muscular atrophy due to zero gravity unless the astronaut exercises regularly. Wisdom89 (T / C)
I think it's been demonstrated for mice though. IIRC, they were born a bit smaller than normal but otherwise okay. DMacks (talk) 02:49, 11 March 2008 (UTC)[reply]
According to Canadian astronaut Chris Hadfield you can't burp normally in a weightless environment - see Q2 here. Gandalf61 (talk) 09:49, 11 March 2008 (UTC)[reply]
Have anyone heard about sex in space? Considering that the blood does not flow normally in space and some parts of our body will need it, perhaps it is not possible... —Preceding unsigned comment added by 217.168.3.246 (talk) 03:03, 13 March 2008 (UTC)[reply]
Blood flows due to heartbeats and one-way valves, even in zero gravity. However, it does stop flowing at high g's, such as those experienced by a fighter pilot. A special suit (I forget the name) can help in this situation, by alternately compressing and releasing on the legs to push the blood forward. I believe sex would work just fine in space, although both parties would want to be contained in an absorbent bag, so that any fluids released don't go floating off down the hall. As a practical matter, though, the lack of privacy and not wanting to use up precious water to clean up afterwards make this a no-no for short space flights. Long-term or even multigenerational ships would need to find a way to make this work, however. I suspect that artificial gravity, via a rotating spaceship, would be used in such a situation, as some gravity seems important in maintaining the human body over the long-term. StuRat (talk) 15:16, 13 March 2008 (UTC)[reply]
Wikipedia knows all - see sex in space. Gandalf61 (talk) 15:24, 13 March 2008 (UTC)[reply]

One man's meat is another man's poison

Are there any spices with an unusually small difference between the standard amount used for flavoring and the minimum poisonous amount? Or with an unusually large difference between the LDLO and the LD50? --67.185.172.158 (talk) 21:11, 10 March 2008 (UTC)[reply]

I seem to rememeber that 2 nutmegs could be fatal...87.102.94.48 (talk) 22:21, 10 March 2008 (UTC)[reply]
I had to look these up: LDLo = Lowest published lethal dose and LD50 = Median lethal dose. Mmmm, Eggnog with extra nutmeg! :-)) --hydnjo talk 01:51, 11 March 2008 (UTC)[reply]

Deadly meat - see fugu. 80.0.108.245 (talk) 19:32, 16 March 2008 (UTC)[reply]

Molecular Excitement

I know that molecular excitement (vibrations, oscillations - what is the correct term?) slows down as Temperature decreases - I'm thinking of a liquified gas in a tank - and increases as the Temperature rises, but what happens when the Pressure increases or decreases? What effect does Pressure have on Molecular Excitement?

P. Lamont —Preceding unsigned comment added by 72.39.249.249 (talk) 21:18, 10 March 2008 (UTC)[reply]

Temperature has the greatest effect on molecular rotation, pressure also influences this, through pressure broadening.--Shniken1 (talk) 22:54, 10 March 2008 (UTC)[reply]
Pressure can change the temperature, if you increase pressure, the substance is likely to contract, and work is done on it. This will add energy, and the temperature will increase. This head can be conducted out. Also if pressure increased so that the material is compressed to a white dwarf density, then the uncertanity principle will ensure a uncertain temperature and Degenerate matter exists. Graeme Bartlett (talk) 23:15, 10 March 2008 (UTC)[reply]

March 11

type 2 diabetes and hair loss

This question has been removed. Per the reference desk guidelines, the reference desk is not an appropriate place to request medical, legal or other professional advice, including any kind of medical diagnosis, prognosis, or treatment recommendations. For such advice, please see a qualified professional. If you don't believe this is such a request, please explain what you meant to ask, either here or on the Reference Desk's talk page.
This question has been removed. Per the reference desk guidelines, the reference desk is not an appropriate place to request medical, legal or other professional advice, including any kind of medical diagnosis or prognosis, or treatment recommendations. For such advice, please see a qualified professional. If you don't believe this is such a request, please explain what you meant to ask, either here or on the Reference Desk's talk page. --~~~~
--Milkbreath (talk) 01:54, 11 March 2008 (UTC)[reply]
Seriously, you need to consult a doctor about this - we can't offer medical advice of any kind... Wisdom89 (T / C) 01:47, 11 March 2008 (UTC)[reply]

Electron configuration in Lawrencium

Lawrencium states [Rn] 5f14 6d1 7s2, while Atomic orbitals#Orbitals table says that "This table shows all orbital configurations up to 7s, therefore it covers the simple electronic configuration for all elements from the periodic table up to Ununbium (element 112) with the exception of Lawrencium (element 103), which would require a 7p orbital."

One of these pages is wrong, is it the atomic orbitals page? Also the next electrons in [Rn] 5f14 6d1 7s2 go into the 6d orbital not 7p... How did the atomic orbital page come up with this statement?

PS: "Simplest electronic state" = ground state? --Shniken1 (talk) 03:17, 11 March 2008 (UTC)[reply]

re: " "Simplest electronic state" = ground state? " - you might think so - but it doesn't have to be the case - because 'simplest' doesn't have much meaning in this context.87.102.14.194 (talk) 08:54, 11 March 2008 (UTC)[reply]
It shouldn't be different here, electron configurations of elements are always given in their ground state, simplest configuration may refer to filling the d orbitals before the next s orbital but that would still leave lawrencium without an 7p electons.--Shniken1 (talk) 10:08, 11 March 2008 (UTC)[reply]

elements

I came across this in a magazine 1. element present in all organic compounds 2.alkane hydrocarbon with straight chain 3.heaviest alkaline earth metal 4.produced by combustion

it seems that 1 and 2 are not elements. but what are the answers? juz curious. not homework.

Uh huh. Anyway, this is an encyclopedia, and you can find answers to simple questions just by typing the relevant word into the search bar. But I'll give you the links right here: Organic compound makes the answer pretty obvious; there are many alkane hydrocarbons that have straight chains; alkaline earth metal, also makes it pretty obvious; combustion, the very first chemical equation in the article gives it away, and I'm guessing you're homework magazine is only concerned with the combustion of hydrocarbons, so that one is all you need. Someguy1221 (talk) 08:39, 11 March 2008 (UTC)[reply]
1. try organic compound
2. read alkane see subsection linear alkanes
3. read alkaline earth metal
4. read combustion87.102.14.194 (talk) 08:52, 11 March 2008 (UTC)[reply]
Has the Physics magazine guy gone to chemistry class now? bibliomaniac15 00:22, 12 March 2008 (UTC)[reply]

The Speed Of Dark......?

I was asked a question recently that sounds nonsense but may have some validity in it. "If light travels at the speed of light, what does dark travel at?" We're told light travels and dark is the expulsion of light, but does it travel? and, if so, at what speed? ajcree —Preceding unsigned comment added by 89.241.133.221 (talk) 08:29, 11 March 2008 (UTC)[reply]

Dark is the absence of light, and nothing more. If it takes t seconds for light from a bulb to reach the wall of your room after lighting up, then it takes those same t seconds for the wall to darken after the bulb dims. Someguy1221 (talk) 08:33, 11 March 2008 (UTC)[reply]
Exactly, so if a star suddenly shut off that it 50 light-years away from us, it would take us 50 years to see that the star had stopped shining. Hope this helps. Thanks. ~AH1(TCU) 16:03, 11 March 2008 (UTC)[reply]
One complication is that objects rarely stop giving off light instantly. The filament from an incandescent light bulb takes a portion of a second to cool to a temp that no longer produces visible light, for example. A star which has ceased to undergo nuclear fusion may take millions of years to cool to the point where it no longer gives off light. StuRat (talk) 17:40, 11 March 2008 (UTC)[reply]
Dark "travels" at the same speed as light because light is just photons, which are perceived using receptors in our eyes. When there are no more photons, we see darkness. Since photons travel at the speed of light by definition, the absence of photons behind them (i.e. dark) expands at the same speed. It's like dropping a stone in a pond: your question is equivalent to asking, "What speed does the trough between the ripples travel at?" Obviously, it travels at the same speed as the ripples themselves! « Aaron Rotenberg « Talk « 09:01, 13 March 2008 (UTC)[reply]

Why do Rabbits hop?

The question is pretty self-explanatory, but please don't answer "because that's how their legs work"! I know that, my question is why do rabbits hop instead of walk like most other mammals. 71.57.90.83 (talk) 08:47, 11 March 2008 (UTC)[reply]

I'd say it's about propulsion and desired speed, and like it or not the structure of both sets of legs being very unlike "most other mammals". Only Bugs Bunny seems to saunter, stroll, tap dance and so on as an upright. Similar to kangaroos (well, not quite), rabbits don't hop – as in having all four peds off the ground – if they are mooching about eating. And as prey animals of that kind, when they need to speed at a moment's notice, the bounce comes into it due to the configuration of the hind legs. Does that help?Julia Rossi (talk) 10:32, 11 March 2008 (UTC)[reply]
Consider a similar animal without the long hoppy back legs - ie a guinea pig if you've ever seen one run you might wonder why they weren't long ago predated out of existence... 87.102.14.194 (talk) 10:42, 11 March 2008 (UTC)[reply]
Interesting point – it probably doesn't exist in the wild any longer. Julia Rossi (talk) 11:11, 11 March 2008 (UTC)[reply]
The article sort of confirms that - seems they use safety in numbers - including random scattering when a predator approaches.. In general though - in a pack of 100 even a 100% efficient predator will only make one kill before stopping I would guess - all that remains is for the creatures to breed sufficiently fast to cover the predation... Poor little things.87.102.14.194 (talk) 12:16, 11 March 2008 (UTC)[reply]
Like when you're being shot at: think zigzag. It's a good way to confuse and loose a predator. 200.127.59.151 (talk) 14:20, 11 March 2008 (UTC)[reply]
...which is useful if the predator is tight on your tail. StuRat (talk) 17:45, 11 March 2008 (UTC)[reply]
I would think that the rabbit's hop is a result of what sort of animals prey upon it. The fox and the hawk come to mind. These are both very quick animals, so the rabbit must also be quick. Its large back legs enable the rabbit to spring forth in an instant, while a more controlled gait would see it end up as lunch more often than not. Vranak (talk) 15:29, 11 March 2008 (UTC)[reply]
Well, if we're just throwing out conjecture, it seems to me that hopping would have evolved in an open environment, grassland maybe, where you won't get snagged in the brush mid-hop or bean yourself on a branch. It is a quickness thing, isn't it? It would have the additional advantage that it would carry you above the grass with each hop, so you could get your bearings. Lots of evolutionary adaptations seem odd, and perhaps this one initially evolved especially to meet the requirements of some small niche, and when the animal spread out it took the form of locomotion with it, retaining it with refinements. It certainly is one good way to make tracks, strange as it seems. --Milkbreath (talk) 19:51, 11 March 2008 (UTC)[reply]
Animals also make do with what they've already got. The ancestor of rabbits may have had legs adapted for something else entirely that turned out to be easy to adapt to hopping and zigzagging, which proved to be a very effective escape. Some gazelle have also evolved a leaping zigzagging flight, but since they started out with a different form of locomotion, they ended up with a somewhat different result. — kwami (talk) 01:05, 12 March 2008 (UTC)[reply]
Why not some OR? Wandering in a wild field in the evening, spotting young bunnies way off, in grasses up to my hips, when a big (medium dog size) rabbit ran past me parallel to the ground (no arching bounces for reconnaissance) very fast and low. And straight. So those back legs have a strong thrust forward when needed, not just the up and down thing. I'm sure it would have zigzagged if it was being chased. Julia Rossi (talk) 03:46, 12 March 2008 (UTC)[reply]
Yes, two different strategies can be employed depending on if the predator is "out of range". If so, running straight and fast to safety makes sense. If the predator would catch the prey before they can reach safety, however, then taking evasive action is the best plan. Zigzagging side to side, hopping up and down, and suddenly reversing direction are all methods that offer some hope that the prey can outmaneuver the predator. Similar methods can be employed by ships, submarines, or planes which are being hunted. StuRat (talk) 14:51, 13 March 2008 (UTC)[reply]

What is this equation?

Edit (hit enter before asking question!) What is this eqation meant to represent? [17] "Pe + (Pe)r" not NaCl....:D --Shniken1 (talk) 10:19, 11 March 2008 (UTC)[reply]

It says 'Pepper' - otherwise it seems meaningless87.102.14.194 (talk) 10:30, 11 March 2008 (UTC)[reply]
"Pepper, not Salt." Someone likes puns. JohnAspinall (talk) 18:27, 11 March 2008 (UTC)[reply]

Why not swing?

Reflecting on the question about rabbits in motion, it set me wondering how come, as descendants of great apes now walking upright, humans don't swing from things as a means of locomotion? Julia Rossi (talk) 10:42, 11 March 2008 (UTC)[reply]

Lack of trees.. imagine if we did though .. we'd have to build adventure playgrounds instead of footpaths.. Seriously though we never would have left the forest.87.102.14.194 (talk) 11:05, 11 March 2008 (UTC)[reply]
First, humans are not descendants of great apes - we are great apes, because we are members of the biological family Hominidae (along with chimapnzees, gorillas and orangutans). And not all of the other great apes swing from trees - gorillas are ground dwelling. As the contributor above says, it is a matter of different species adapting to different environments. The same thing happens in the squirrel family, where we see both tree squirrels and ground squirrels. What exactly caused humans to acquire bipedalism is still a matter of research and debate - our bipedalism article says "There are at least twelve distinct hypotheses as to how and why bipedalism evolved in humans". Gandalf61 (talk) 12:37, 11 March 2008 (UTC)[reply]
I'd go with the idea that as prehumans moved out onto prairie, which lacked trees, they needed to develop an efficient method of locomotion for that environment. Since many primates can stand on hind legs for short periods, it wasn't too much of a leap to develop full-time bipedal motion. While less efficient than quadruped motion, bipeds had the huge advantage of having their hands free to use tools and weapons, which the opposable thumb our primate ancestors developed for grasping tree limbs first made possible. The relatively high intelligence of primates also made it possible to put tools to good use. StuRat (talk) 17:57, 11 March 2008 (UTC)[reply]
But lots of primates have moved out onto the savanna, and they didn't go bipedal. Also, Lucy apparently wasn't much of a tool user, but she was bipedal. All these arguments about what makes us different from other apes - we lost our hair cuz it was hot, we stood on our hind legs to see further or to free our hands - remind me of speculations about the extinction of the dinosaurs back before the extra iridium was discovered in the K-T boundary layer: They all failed, because they didn't address why, for example, plankton should go extinct just because dinosaurs got constipated from flowers (yes, I think that was actual proposal!). — kwami (talk) 00:59, 12 March 2008 (UTC)[reply]
There were multiple niches for primates on the savannah, some of which were for bipedal primates. Being a quadruped would have other advantages, like being able to hide from predators and prey in tall grass and running more quickly. As for being a tool user, I expect that the earliest tools were just found objects, like sticks and rocks, so I wouldn't expect to find a collection of chipped flint tools nearby. StuRat (talk) 15:16, 12 March 2008 (UTC)[reply]
Back to the original question, humans have retained some ability to climb trees, which is why we have arches in our feet (to fit the tree trunk). StuRat (talk) 17:57, 11 March 2008 (UTC)[reply]
I have to say, I'm extremely dubious of the "arches are for climbing trees" idea. Arches of the foot are very important to humans being able to support their weight and to being able to be bipedal. --98.217.18.109 (talk) 23:48, 11 March 2008 (UTC)[reply]
In that case, the arches would go all the way across the foot, symetrically, not only on the inside edges of the feet. This form is perfect for grasping tree trunks. StuRat (talk) 03:42, 12 March 2008 (UTC)[reply]
Why don't we swing from things? Why should we? Only some of the great apes spend any time swinging from things, and even those spend a lot of time not swinging from things. When not around things to swing from, they don't swing, and get around just fine. Swinging is both difficult and dangerous; it's not an ideal form of locomotion. --98.217.18.109 (talk) 23:48, 11 March 2008 (UTC)[reply]
Given the ground dwelling great apes, how come we lost all that upper body strength belonging to the longer arm ratio that they retain? I'm wondering if maybe it had something to do with the homosapiens taking over from the neanderthals and using the hands for specialisation that swinging might otherwise have interfered with. (oops the third part was answered in the thread where it goes back on itself above - I missed that) Julia Rossi (talk) 03:34, 12 March 2008 (UTC)[reply]
I expect that it has to do with our weak backs. The spine was really not designed to be used as we use it, so can't support nearly as much weight in bipeds as in quadrupeds. This means our upper bodies had to be minimized, such as having shorter, less muscular arms. We also have thinner skulls than many of our quadruped primate friends. If I had to redesign the spine for bipeds, I'd take the nerves outside the spine into something like a notochord, and replace them with cartilage to try to hold the spine together. StuRat (talk) 15:24, 12 March 2008 (UTC)[reply]
Why would we invest in longer, stronger arms when that would interfere with walking and running? Shorter, less strong arms are more convenient for not swinging from tree to tree, so if it's more of an advantage to be walking and running on the savannah than swinging from tree to tree there is a disadvantage in investing energy in those tree-swinging arms. I don't really see what's to explain here :/ Skittle (talk) 18:29, 12 March 2008 (UTC)[reply]
  • Humans stick to the ground because we've grown too tall and heavy to swing from trees. Gorillas suffer the same problem. As you know from Newton's law of universal gravitation, there's an inverse-square relationship at play: . What that means is that the proverb "the bigger they come, the harder they fall" is literally true: a taller, heavier creature hits the earth with much, much more force if it falls. At a certain point, evolutionarily speaking, the behavior is too risky to be worth pursuing. --M@rēino 20:27, 12 March 2008 (UTC)[reply]


I'd just like to point out that some of us do swing. In fact, otherwise, it don't mean a thing. --Trovatore (talk) 20:50, 12 March 2008 (UTC)[reply]

astronomy

the stars in the sky seen from the UK are the same as from Australia?cypru÷90.193.58.40 (talk) 13:51, 11 March 2008 (UTC)alex[reply]

Nope. While there is some axial tilt to the Earth's rotation, the UK sees a great many stars well north of the ecliptic plane and Australia sees a great many well south of the same. Stars near the ecliptic will be visible to both places (though constellations will be inverted) but many regularly visible to one are never visible to the other. — Lomn 14:12, 11 March 2008 (UTC)[reply]
Two good examples would be the North Star and the Southern Cross. AlmostReadytoFly (talk) 14:34, 11 March 2008 (UTC)[reply]
Hi. A good indicator of which stars you can see is to take your latitude, positive if north and negative if south, then add 80 for northern limit and subtract 80 for southern limit. Well, that should be a good indicator of which stars you can see without too much interference from the atmosphere, light pollution, terrestial objects, haze, and all that. So, if for example the southern limit for a location in the UK is -25, then you can see Sirius easily because it is at -17 or so, oh and if you get a number like +120, subtract 120 - 90 to mean that you can see 30 degrees in declination beyond the north pole reigon near the pole star to the north, and similar methods for southern hemisphere. Hope this helps. Thanks. ~AH1(TCU) 16:10, 11 March 2008 (UTC)[reply]

Alcohol injection

Is it possible to survive when a small amount of alcohol is injected intravenously (for instance, a 40% vodka within an average syringe capacity, or less, if lethal)? IMO, one should break through because it's similar to drinking alcohol on an empty stomach. No one is experimenting here:) --85.132.14.38 (talk) 18:54, 11 March 2008 (UTC)[reply]

Injecting a small amount of ethanol and injecting a small amount of 40% vodka are two different things. If I had to choose, I'd choose the ethanol since I don't know what else might be in the vodka. ----Seans Potato Business 21:08, 11 March 2008 (UTC)[reply]
Steve-O did a whole IV bag of vodka intravenously (well, duh!) in one of his videos. It didn't kill him. That's not to say that you should try it at home (or at work, or in someone else's home, or anywhere else for that matter). --Kurt Shaped Box (talk) 00:21, 12 March 2008 (UTC)[reply]
Let's consider the math in the reverse direction. A blood alcohol level of >0.40% is usually fatal, so let's consider a BAC of 0.30% (the point where you'll generally lose consciousness). A typical adult male has about 5 litres of blood. So 0.30% of 5L is 15mL of alcohol. So, if you're using a typical 1mL, 3mL or 10mL syringe, then it looks like you'll be fine you may survive, even if it's full of pure ethanol. However, an IV bag with 40% alcohol all at once would almost certainly kill you. There was a problem a few years ago where people were experimenting with inhaling alcohol (you get drunk way faster, but it's also much easier to take a lethal dose vs. ingested alcohol). My advice: stick to the recommended one-drink-per-day (or less). (EhJJ)TALK 01:01, 12 March 2008 (UTC)[reply]
I should also point out that Steve-O was not 'well' as such after his experiment - it just didn't kill him. His body is probably far more accustomed to abuse than yours or mine too. --Kurt Shaped Box (talk) 01:15, 12 March 2008 (UTC)[reply]
I would not count on being "fine" after injecting 10 ml of absolute ethanol. Total quantity of alcohol in the bloodstream is not the only issue here. You're going to get all sorts of local damage at the injection site, probably lysis of blood cells, God knows what else. (Not to mention that it's going to hurt like anything.) Could it result in a clot that would wind up in your coronary artery? Don't know; don't see any reason to do the experiment. --Trovatore (talk) 01:17, 12 March 2008 (UTC)[reply]
Trovatore's quite right about the concentration effects—pure ethanol does really nasty things to living cells, and I agree with him about the likelihood of damage at the injection site. (Not that I would guarantee 40% ethanol won't do damage, mind you—and case reports indicate that it burns like the devil going in: [18].)
As to EhJJ's calcuation about lethal BAC's, it's worth noting that ethanol is a small molecule, and the concentration of ethanol in surrounding tissue will rapidly come to equilibrium with the ethanol concentration in the blood. This equilibration is a fairly rapid process, so you really need to be looking at a dilution of the injected alcohol into nearly all of the body's water rather than just the circulating blood volume. Even bearing that in mind, of course, a full IV bag is rather a lot of vodka to consume at one sitting, by any route of administration. TenOfAllTrades(talk) 02:22, 12 March 2008 (UTC)[reply]
Indeed, one would need to know the volume of distribution of ethanol. The volume of distribution page actually lists ethanol as an example @ 30L (no citation). Therefore EhJJ's back-of-the-envelope calculations may be off by a factor of six. But I imagine anything 40% or greater bust sting like hell as an injection... — Scientizzle 20:06, 12 March 2008 (UTC)[reply]
That article mentions snorting vodka. I've done that (a dessertspoonful). Not very much fun and not something I'm planning on doing again - it stung and made my eyes water like crazy. I was already drunk at the time, so I can't tell you if it had any significant intoxicating effect. --Kurt Shaped Box (talk) 02:42, 12 March 2008 (UTC)[reply]

Electron and Hole Concentration in Carbon

I've been trying to find electron and hole concentration data for carbon to calculate the Hall coefficient to predict the Hall p.d. across a thin carbon sheet. I can't seem to find any although I managed to find the data for silicon (http://mems.caltech.edu/courses/EE40%20Web%20Files/Supplements/02_Hall_Effect_Derivation.pdf)

Do these values depend on the sample of carbon or is there any other way of predicting the Hall p.d.? Welsh-pingpong (talk) 21:35, 11 March 2008 (UTC)[reply]

What kind of carbon are we talking about here? Graphite, diamond, or some nanomaterial like graphene (which is an atomically "thin carbon sheet")? They all have different energy band structures. Assuming we're talking about a graphite sheet of macroscopic thickness, this paper says the electron and hole concentrations are each about cm−3 at low temperatures. Which brings up another important question: what temperature is the carbon? —Keenan Pepper 23:00, 11 March 2008 (UTC)[reply]
It's a graphite sheet of thickness 0.18mm. The temperature was at room temperature with 0.1A passing through it - I didn't notice any significant temperature increase. How does temperature affect electron and hole concentrations? Also where can I find data about electron and hole mobility - I'm trying to make a prediction of the Hall voltage and need these values for the Hall coefficient. —Preceding unsigned comment added by Welsh-pingpong (talkcontribs) 08:51, 12 March 2008 (UTC)[reply]

March 12

vitamin A

wat way can i chemicaly make or extract pure Vitamin A. And if i can't get it pure, how can i get it the purest? —Preceding unsigned comment added by 76.14.124.175 (talk) 04:03, 12 March 2008 (UTC)[reply]

Probably easy to extract it from a commercial over-the-counter vitamin supplement. Depends what you want to do with it how pure is "pure". DMacks (talk) 06:35, 12 March 2008 (UTC)[reply]
Vitamin A can be make synthetically - you might need to know a lot of chemistry though - a good library should have something about it's synthesis in the organic chemistyr section.
It's possible to buy vit A (Retinol) from a chemical supplier at 99% pure.
It would also be possible to extract vitamin A from a sample containing it - though you would probably need to do chromatography to get it pure.87.102.17.32 (talk) 13:31, 12 March 2008 (UTC)[reply]

CHloroacetic acid

Below is how it is made. But wat are the exacts? do i have to heat it? do i have to use pure chlorine? how do i use the catalyst?

Chloroacetic acid is synthesized by chlorinating of acetic acid in the presence of red phosphorus, sulfur, or iodine as a catalyst:

CH3CO2H + Cl2 → ClCH2CO2H + HCl —Preceding unsigned comment added by 76.14.124.175 (talk) 04:32, 12 March 2008 (UTC)[reply]

I'm gonna go with "if you don't know how to look these kinds of things up in the chemistry literature, you shouldn't be doing this reaction". DMacks (talk) 06:33, 12 March 2008 (UTC)[reply]

Chemisrty literature??????????????? —Preceding unsigned comment added by 76.14.124.175 (talk) 23:48, 12 March 2008 (UTC)[reply]

chemotherapy or radiation therapy for obesity

Could anticancer therapies be tweaked or redesigned to attack fat cells? Thanks, 05:37, 12 March 2008 (UTC)Rich (talk) 05:38, 12 March 2008 (UTC)[reply]

Have you read our article on antineoplastics and/or chemotherapy? There are a number of different techniques, but they typically rely on cancer's fast growth rate (for example, they slow/damage new DNA synthesis). This works really well on fast growing cells (such as malignant tumors and hair follicles), it doesn't typically affect slow growing cells (such as neurons and adipocytes (aka fat cells)). So, generally speaking, I'd say no. (EhJJ)TALK 11:43, 12 March 2008 (UTC)[reply]
Also, it would be bad to get rid of fat cells. You want a lot of small fat cells for smooth skin. With no fat cells you would look like a skeleton, and with a few fat cells they would tend to become large and clump together, giving you a lumpy appearance. StuRat (talk) 15:01, 12 March 2008 (UTC)[reply]
Good answers, thanks.Rich (talk) 21:09, 12 March 2008 (UTC)[reply]

steroids

Whats the difference between anabolics and steroids? —Preceding unsigned comment added by 75.4.67.159 (talk) 05:43, 12 March 2008 (UTC)[reply]

Our anabolic steroid page is a good place to start. DMacks (talk) 06:11, 12 March 2008 (UTC)[reply]

Quantum Mechanics: Entangled Wave Function

The equation 9, or (EPR9) for short here, in the original paper of EPR paradox gives a wave function of two entangled particles

(EPR9)

where is Planck's constant, and are the variables describing the two particles and is just some constant. According to reduction of the wave packet, when an observable of the first particle is measured, (EPR9) can be expanded by the eigenfunctions of in the form

(EPR8)

where are the corresponding coefficients to the eignefunctions. If is a continuous observable, the coordinate of the first particle, (EPR8) can be written as

(EPR15)

According to the paper, the eigenfunctions of is

(EPR14)

which has corresponding eigenvalue . The first question is how come the eignefunction and the eigenvalue of are (EPR14) and , respectively? It seems that

(1)

and if we let

(2)

then

(3)

Find the solution of

(4)

we have

(5)
(6)

the right-hand side of (6) is the eigenvalue of . Similarly, the eigenvalue of the observable

(EPR17)

can be found by knowing

(EPR16)

and let

(7)

The solution of

(8)

is

(9)

Again, the right-hand side of (9) is the eigenvalue of which complies with the paper. But it still doesn't explain how to figure out the eignefunction (EPR14).

To continue the unsolved discussion last time, the second question is how to denote the entangled wave function (EPR9) in bra-ket notation? If it can be done, it should help with respect to the last discussion. The bra-ket notation of (EPR9) is supposed to be in the Hilbert space which is the tensor product of the state spaces associated with the the two particles. - Justin545 (talk) 06:36, 12 March 2008 (UTC)[reply]

Hi, I'm sorry I haven't followed up to the old thread yet, but maybe a response here will serve the same purpose.
There are many ways to write (EPR9) in bra-ket notation; for example I could just write where Ψ is defined by (EPR9). In terms of tensor products of kets inhabiting the state spaces of the individual particles, I could write for example . I'm not sure those are properly normalized, to the extent that these mathematical monstrosities can be considered to be normalized to begin with. The product might also equivalently be written or or . The subscripts 1 and 2 just indicate which subspaces the kets inhabit; they could be left off since the two subspaces are isomorphic in this case.
I'm not sure I understand your first question. Finding eigenfunctions of the position operator in a single-particle space involves solving equations of the form where B(z) = z and BΨ is a pointwise function product. It should be clear enough that the only possibilities for Ψ here are functions that are zero everywhere except at a point, and the "normalized" versions of these functions are the delta functions, which form an orthonormal eigenbasis. In the two-particle space things are a bit more interesting. You're now solving where . The normalized solutions here are where g is any normalized function of x2. These do not form a basis; there are far too many of them for that. You have to choose arbitrarily some orthonormal basis for the functions g. This happens because there are degenerate eigenvalues; the discrete analogy is that there's only one orthonormal eigenbasis for diag(1,2,3) but many for diag(1,1,2). -- BenRG (talk) 12:55, 12 March 2008 (UTC)[reply]
It's reasonable making the ket be the function of the corresponding eigenvalue since each eigenvalue identifies an unique basis or eigenfunction. But, I am a bit confused with the bra-ket notation since I expect the bra-ket notation should be in the form
(10)
rather than in the form
(11)
It seems the integral surrounding the ket can not be removed. But, will the integral of the ket yield another "ket" in the same space? Another confusion is about the momentum part of the bra-ket example . I am not able to figure out in it.
Apologies for obscuring my first question. My first question is just to understand why the eigenfunction of is a "delta function". Just wonder how the delta function (EPR14) is mathmatically derived. As you said "Finding eigenfunctions of the position operator in a single-particle space involves solving equations of the form where B(z) = z and BΨ is a pointwise function product." But I can not understand why it's pointwise. Excuse my poor quantum mechanics, I left so many question marks here :-) Justin545 (talk) 08:43, 13 March 2008 (UTC)[reply]

Cancer

Why do swelling often occurs around the tumor? there doesn't seem to be an answer in wikipedia.


There's no single answer; tumor can block lymphatic drainage and cause fluid accumulation; cytokines secreted by tumor cells can cause inflammation; some tumors upregulate angiotensin II type 1 receptors, triggering chronic inflammatory response; some tumors cause the formation of new blood vessels (angioneogenesis) causing hyperemia and swelling. - Nunh-huh 06:59, 12 March 2008 (UTC)[reply]
And then there's the simple method of the tumor growth itself pushing on the surronding tissue, which then expands outward, similar to how a growing weed can push asphalt upward. StuRat (talk) 14:57, 12 March 2008 (UTC)[reply]
Tumors often elicit immune responses from cytotoxic T lymphocytes, but also bear in mind that tumors promote angiogenesis as well. Wisdom89 (T / C) 18:49, 12 March 2008 (UTC)[reply]

Hepatitis C

How dangerous and contagious is hepatitis C, can it be transmitted through a cats claws? —Preceding unsigned comment added by DSTiamat (talkcontribs) 08:31, 12 March 2008 (UTC)[reply]

If you'll read our article on Hepatitis C, you'll find that it is spread by blood-to-blood contact, and over 90% of cases are spread through blood products or recreational drug use. A less common form of spread is via sexual contact. Though one can make up scenarios in which an infected person is scratched by a cat, who then immediately scratches and infects someone else, this is not something that's ever been reported as actually happening. Hepatitis C infections can range from asymptomatic to fatal, so obviously its dangerousness varies greatly, depending on a number of other factors, including luck. - Nunh-huh 09:10, 12 March 2008 (UTC)[reply]
If you want a disease associated with cat scratches, look up cat scratch fever. StuRat (talk) 14:52, 12 March 2008 (UTC)[reply]

Who makes the 120mm fans used on the space shuttle?

I have no Idea on where to go to start looking for such information, how many different manufacturers are used and what they are used for...just think it'd be interesting. —Preceding unsigned comment added by 71.237.205.31 (talk) 09:11, 12 March 2008 (UTC)[reply]

You need to start with a specific module so you can narrow down to the country that built the module. Then, you can hope some of the contracts were made public within that country. -- kainaw 15:16, 12 March 2008 (UTC)[reply]
The shuttle has at least four different types of fans in the environmental control system. There are two cabin fans, six avionics bay cooling fans, three IMU fans, and at least one airlock booster fan; there may be more I'm forgetting. I'm not sure if they are all identical (I would assume not). I don't know any details (or even where to start looking) about make and model of the fans. anonymous6494 18:32, 12 March 2008 (UTC)[reply]

Biochemistry

The process in purin purification by Molecular exclusion62.24.99.237 (talk) 10:51, 12 March 2008 (UTC)[reply]

You probably meen purine and "molecular exclusion" can be searched for on the web, or may be described at Size exclusion chromatography87.102.17.32 (talk) 18:40, 12 March 2008 (UTC)[reply]
Could you be more specific87.102.17.32 (talk) 18:41, 12 March 2008 (UTC)[reply]

Panic Attack while teaching my kid

This question has been removed. Per the reference desk guidelines, the reference desk is not an appropriate place to request medical, legal or other professional advice, including any kind of medical diagnosis or prognosis, or treatment recommendations. Please consult a physician, psychiatrist or psychologist--(EhJJ)TALK 11:33, 12 March 2008 (UTC)[reply]

(EhJJ)TALK 11:33, 12 March 2008 (UTC)[reply]

heat conductor

Why does the a plain cloth catches fire when put near a flame at about 10 cm compared to the cloth wraping a coin in it? —Preceding unsigned comment added by 165.21.155.69 (talk) 12:43, 12 March 2008 (UTC)[reply]

Maybe it gets hotter quicker without the coin?87.102.17.32 (talk) 13:33, 12 March 2008 (UTC)[reply]
Yes, the coin is a good thermal conductor, so pulls heat out of the cloth, acting as a heat sink, until it gets up to temp, then it will no longer draw off heat. StuRat (talk) 14:49, 12 March 2008 (UTC)[reply]

we are looking for info regarding mining

What is fools gold ( piriet) and why do they call it fools gold?

The impact that the mining of gold has on SA's environment: The use of sinade and uranium as by-products in the mining process The influence of mining on water and the effect of water on buildings Mine heaps amounts of rock removed from the earth surface. Air Pollution. —Preceding unsigned comment added by 196.211.98.154 (talk) 13:03, 12 March 2008 (UTC)[reply]

Fools gold is pyrites (spelling) it's actually Iron Sulphide.
sinade = cyanide which is used to extract the gold.87.102.17.32 (talk) 13:13, 12 March 2008 (UTC)[reply]
Gold mining should be of interest. there's some info on the enviromental hazards of cynaide seeCyanide_process#Effects_on_the_environment.87.102.17.32 (talk) 13:16, 12 March 2008 (UTC)[reply]
Also try http://www.google.co.uk/search?hl=en&sa=X&oi=spell&resnum=0&ct=result&cd=1&q=gold+mining+environment&spell=1
Looks like uranium is a by product and a pollutant produced when gold in mined http://www.ingentaconnect.com/content/klu/gejo/2004/00000061/00000002/00002867;jsessionid=1rj6eslojhj0m.alexandra? —Preceding unsigned comment added by 87.102.17.32 (talk) 13:20, 12 March 2008 (UTC)[reply]

Looks like South African homework. :-P --98.217.18.109 (talk) 14:29, 12 March 2008 (UTC)[reply]

Hi. It might be. OP is from South Africa. ~AH1(TCU) 00:48, 13 March 2008 (UTC)[reply]

Cuttlefish wants to cuddle

How do Cuttlefish know how to blend in with a given environment if they can't see in color? Why would their skin have such a wide range of colors possible if they can't distinguish between most of them? --98.217.18.109 (talk) 15:07, 12 March 2008 (UTC)[reply]

According to our article, cuttlefish change color to camouflage themselves as well as communicate. The camouflage function would benefit from a range of colors even if they are colors the cuttlefish can't themselves see. As for how they match colors they can't see, I don't know how they do that, and according to this article, at least one cuttlefish camouflage expert doesn't know either. --Allen (talk) 15:33, 12 March 2008 (UTC)[reply]
This article is also fascinating. —Keenan Pepper 18:37, 12 March 2008 (UTC)[reply]
See also chromatophore, specifically the section on Cephalopod chromatophores. Rockpocket 19:00, 12 March 2008 (UTC)[reply]
Let's get this straight -cuttlefish are colour blind but they do match themselves to the colour of the surroundings??? so they camoflage based on the grey scale image they see —Preceding unsigned comment added by 87.102.17.32 (talk) 20:55, 12 March 2008 (UTC)[reply]
Well, you're assuming that the color change is in response to a visual signal. Maybe it's not; maybe something in the cuttlefish's skin (if that's the right word) responds directly to the color of the light hitting it, without going through the eye at all. --Trovatore (talk) 21:04, 12 March 2008 (UTC)[reply]
I wondered about light sensors in the skin..
Though in one of the references above -(nytimes) - it describes the placement of a high contrast pebble in the fish tank, the cuttlefish then 'looking at it' and then adding a black spot to its skin pattern.. That shows (I think) at least that the eyes are at least a part of the camoflage set-up..87.102.17.32 (talk) 21:50, 12 March 2008 (UTC)[reply]

I hadn't read the Science article when I gave my answer; now I've scanned it and it looks like the NYT reporter Carl Zimmer must have made an error. He quotes Dr. Hanlon as saying "we don't know how" [in Zimmer's words:] "they see a world without color, but their skin changes rapidly to any hue in the rainbow." But the Science article clearly demonstrates that cuttlefish's skin does not change to any hue in the rainbow. So maybe there's no mystery: cuttlefish are color blind, and therefore have trouble camouflaging themselves against certain colorful backgrounds. But on the other hand, there is the last sentence of the Science paper: "However, the vexing question of how S. officinalis masters the task of camouflage in chromatically rich environments, such as those found at shallow depths of water, remains to be answered." So Mathger et al. are saying there is a mystery... but what are they referring to? The paper doesn't seem to cite any instances of S. officinalis mastering the task of camouflage in any truly chromatically rich environment. At least not that I can see. --Allen (talk) 02:21, 13 March 2008 (UTC)[reply]

urea formation

Hi all read the article on urea cycle found it a bit confusing... could anyone tell me how the NH2 group removed from an amino acid in deamination is converted to ammonia (NH3), Where does this hydrogen come from? I believe the NH2 is converted to NH3 before it reacts with CO2 to form urea, but please correct me if that is wrong. Thanks. —Preceding unsigned comment added by 172.142.47.24 (talk) 21:10, 12 March 2008 (UTC)[reply]

Are you refering to "conversion of glutamate to ammonium and α-ketoglutarate." - the 'co-factor' is NAD+ see Glutamate dehydrogenase. Was that the bit you meant. the amino acid is oxidised to a keto compound.
In the big diagram http://en.wikipedia.org/wiki/Image:Urea_cycle_2.png 'ammonia' remains bound to the molecule as an iminium cation, with loss of fumarate, water hydrolysis the iminium cation to urea..87.102.17.32 (talk) 21:42, 12 March 2008 (UTC)[reply]


Urea Cycle Metabolic Pathway
L-citrulline Carbamoyl
phosphate
L-ornithine
File:L-citrulline wpmp.png Pi File:Carbamoyl-phosphate wpmp.png File:L-ornithine wpmp.png
File:Biochem reaction arrow special 3.png
L-aspartate Urea
File:L-aspartate wpmp.png + ATP File:Biochem reaction arrow special 1.png File:Biochem reaction arrow special 2.png File:Urea wpmp.png
PPi + AMP H2O
File:L-argino-succinate wpmp.png
File:Biochem reaction arrow special 5.png
L-argininosuccinate Fumarate L-arginine


This is probably the easier diagram - if you are still stuck can you reference your questions to this diagram. Just to make it easier (assuming it's relevent)87.102.17.32 (talk) 21:45, 12 March 2008 (UTC)[reply]

The "fish scales" in the clouds are back!

pink-teal iridescence, traces of hints of blue, violet, orange
pink-teal iridescence, orange-red halo, blue, green, and violet patches, blue-green inner halo glow, near-sun whiteness

Hi. Remember my previous question? The one a few weeks ago with the 8 deg halo and the iridescent "fish scales" in the clouds? Well, here's the story. Yesterday I saw a roughly 25 deg from the sun rainbowlike halo, and a sundog while the sun was in a cirrus-cirrostratus-altostratus cloud. Anyway, remember last time I saw those "fish scales" in the clouds and that halo as I was typing? Well, I didn't have a camera. Well, guess what? This time I was in a car. I saw it again. At first I thought we didn't bring the camera, but it turns out we did. Yay! I took dozens of pictures, but erased every one of them except seven, because the others were too indistinct. Anyway, I uploaded two of them. Click on the images if you wish. Now, remember last time there was a cirrocumulus cloud passing by as I saw the iridescent fish scales? This time, cloud after cloud passed by or near the sun. The way I saw them were much, much more colourful than in the pictures, probably because the camera was automaticly set to neutral colour or something. Anyway, the clouds were probably stratocumulus, but some of them were very fibrous, and there were some clouds slightly higher than others. Look carefully for the colours, because they're much harder to see in the images than in real life, which is why I only picked the best two. Notice in the pictures: The organge halo, the pink-teal iridescence, and a hint of rainbow colours. What do you think they are, and do we have an article on them? Thanks. ~AH1(TCU) 23:23, 12 March 2008 (UTC)[reply]

It's not exactly a glory, but that page may help. Cheers Geologyguy (talk) 01:01, 13 March 2008 (UTC)[reply]
No, it's not a glory; I guess the closest article would be Cloud iridescence. It's simply dispersion of sunlight by water droplets in the clouds. FiggyBee (talk) 03:43, 13 March 2008 (UTC)[reply]
There's a nice sample here[19] at the Cloud Appreciation Society site, but I like your "fish scales" description. Your pics wouold go well in the iridescence article to relieve the high contrast ones it has, to show some alternatives. I've seen what you describe – glad to know it's a phenomenon. Julia Rossi (talk) 07:24, 13 March 2008 (UTC)[reply]
Wow a RARE PHENOMENON!!! I saw this phenomenon twice and I had my camera one time and uploaded it!!! YAY!!! ~AH1(TCU) 17:17, 13 March 2008 (UTC)[reply]
You might check out Marcel Minnaert's book: The Nature of Light and Color in the Open Air. --Carnildo (talk) 21:40, 13 March 2008 (UTC)[reply]

March 13

Recent rise in oil prices?

Hi. I'm pretty sure most of you have noticed recent sharp increases in oil prices. In fact, just a few years ago, there were occasional times where some gas stations would give prices of less than 50¢ a litre. At that time, gas prices per litre seldom reached above 90¢ per litre. Prices above $1/l were almost unheard of. Nowadays, prices above $1 are the norm. Prices below 90¢ are now almost unheard of. The news has said that, this spring, prices could reach $1.30. Oil is also constantly above $100 a barrel, if you're American. What's going on? This all started after hurricane Katrina, it seems. Are we reaching global peak oil already? Yes, I know the US and non-OPEC countries have already peaked. Are oil companies doing this on purpose? Or is this permanent and a sign of what's to come? What's next? $2.00 a litre within the next decade? $200 a barrel? Would peak oil cause a spike in prices? Yes I know it will probably run out completely in like, 40 years, so hopefully we switch to non-fossil fuels by then. As oil dwindles but not completely, what will we see? Maybe $10 a litre, $900 a barrel? Thanks. ~AH1(TCU) 00:03, 13 March 2008 (UTC)[reply]

Your question is asking people to foretell the future. This is a reference desk, not a magic crystal you can rub and hope for a good answer. However, it isn't hard to figure out. Oil supplies are drying up. Oil demand is increasing faster than ever before. What do you think the price should do? -- kainaw 00:13, 13 March 2008 (UTC)[reply]
The present run-up (that is, above US$95 or so) is mostly related to the weak dollar - people want to hedge their investments by moving them to commodities. Daily fluctuations ($2-$3 or thereabouts) usually reflect particular news items, from weather to stocks reports to saber rattling. The basic price (say something like US$85 to $95) is a reflection of supply (near or at capacity right now) and demand (increasing despite the price). Best get used to it. Cheers Geologyguy (talk) 00:20, 13 March 2008 (UTC)[reply]
Any of your scenarios are possible, if not likely, but who can tell the future. For example I've heard serious predictions of $2 - $3 per litre within the next few years in Australia (current average price is about $1.40). And many sensible commentators also suggest that we've already passed worldwide peak oil and are now on the decline. --jjron (talk) 13:49, 13 March 2008 (UTC)[reply]

I find it entertaining seeing Americans complaining about fuel prices when we pay £1 per litre (which is about $8 per gallon) in the UK TheGreatZorko (talk) 14:11, 13 March 2008 (UTC)[reply]

Most of that is tax though. But I will say, complaining is fun, and it allows us british to feel superior knowing that we have much more right to complain than they do. -mattbuck (Talk) 14:30, 13 March 2008 (UTC)[reply]
Speaking as an American who is not complaining (mostly I'm amazed that our prices are not a lot higher), I really shudder to think what the US will be like when reality actually sets in. Cheers Geologyguy (talk) 14:34, 13 March 2008 (UTC)[reply]
I don't complain either because I don't drive an SUV the size of Kansas around. One time I pulled up to the pump after the previous person (who was an owner of a huge SUV) left and almost laughed to death when I saw he had paid almost $100 for gasoline. I haven't been to Europe in about 5 years, but from what I remember they aren't as stupid as we are, and they buy smaller cars unless they need to tow something. Do get me started on driving habits either - everyone I see with a superfluous SUV wastes gas by flooring the pedal the moment the light turns green. And yet they complain, and then they wonder why the world hates America so much. 206.252.74.48 (talk) 19:33, 13 March 2008 (UTC)[reply]

climate modelling simulation map-based continental land-influenced scenario?

Hi. I have a method you might be able to use for calculating the climate of a specific time period. Look at it and tell me what you think and what adjustments might need to be made if it plausibly works. Here are the steps:

map
  • first you need to draw on a piece of paper the shape of your Earth. Make sure you include the whole world in your map shape. You may want to get a huge piece of paper, and maybe several pieces. Now draw a grid for your map. Next, draw the continents. Take note of the elevation. If it is a map far into the past or future by millions of years, make sure you examine all the evidence. Make four maps, one for mid-winter, mid-spring, mid-summer, and mid-autumn. The seasons may not be at the months they are today because of precession, or the days might be longer or shorter, so don't write the month. If this is a period without seas, water, water vapour, sunshine, oxygen, vegetation, volcanoes, or plates, or if it is during a great bombardment, the map will be useless for that era.
  • Examine the plates at the time of the era of your map. If the fossil record shows proof of high mountainous habitats or undersea environments, include it in your map. Do not base the continents on what they look like today. Take note of the sea level. If the sea level was 300 metres below today's, then treat a piece of land that would be at sea level today as if it was 300 metres above sea level, and same if the sea level was or will be higher. If you're doing to future, include as many factors as possible in your simulation of placement of the continents. Draw in any plausible islands. If the mountain ranges that are here today did not exist back then, do not draw them. If it is likely that where there are mountains today were part of the sea floor back then, treat it as the sea floor. Make note of what type of climate it was. If there is evidense of a gulf or bay or inland sea or lake, draw it in. If there are landmasses near the poles there will likely be large glacial sheets.
  • make notice of the rivers and glaciers. If the mountains were high and cold enough, draw in glaciers at that reigon. If it is likely that there were pack ice or large sheet of ice like on Antarctica or Greenland today, draw it in. Make sure you draw these at the correct portion of the grid. If there was a seamount or undersea volcano or likely there was a coral reef or will be and the sea level is low enough to hold them above sea level, draw that in. Calculate sea level carefully, take in the factors. Look for likely positions of rivers and wetlands. If you know that there were deserts or forests in a particular location or will be, make not of that.
jet stream
  • draw 6 seasonal jet streams for both the northern and southern hemispheres, 2 polar, 2 temperate, and 2 tropical. Do this for each season. If the season in the hemisphere is summer, draw the jets closer to the poles, farther in winter, and moderately close tot he poles in spring and moderately far in autumn. Assuming that the Earth is rotation west-east during this era. Draw the polar jet streams with wind from east to west. Draw them from east to west, and make sure they meet on both ends of the map, like if it was at 80 deg at the rightmost egde of the map, make sure it is at 80 deg at the leftmost edge of the map. Cause it to dip south if there is a mountain range or large glacial sheet. If it was a warm period, draw them close to the poles. If it was a cold period, draw them away from the poles. If it was an ice age or there is an ice sheet on land kilometers thick covering the entire polar reigon and then some, draw them from west to east. Make sure that if there is a huge sheet of ice like this, make it so that the jet streams avoid the ice. On the leeward side of large tall mountain ranges that polar jet streams cross, which is the side facing away from the jet streams, cause the streams to dip towards the poles. The streams dip away from the poles if it encounters tundra or desert or ice, toward the poles if it encounters unfrozen water. It will dip towards the poles when it encounters forests in spring or summer, and away form the poles in autumn or winter. It will dip towards the poles if it encounters a coastal area after being over the sea.
  • the temperate jets are at temperate latitudes, from west to east Again draw them according to the season and warmth of the period. If it encounters mountains it will dip toward the equator. If in a warm season it encounters water it wil dip away from the poles and if in a cold season it will dip toward the poles. Rivers don't count as "water" unless it was a bay or gulf hundreds of km wide. Large areas below sea level but are not covered in water get a poleward dip and for other jets as well. Wetlands act like bodies of water if they are large enough. Deserts get poleward dips in spring and summer and equatorward dips in autumn and winter. The dips in spring and autumn are less than in summer and winter. Dip equatorward for any glaciers. For dry salt pans dip equatorward.
  • tropical jest go from east to west. Again draw them according to the global climate at the time. Oceans get a dip toward the equator, but if it just came from the sea and just met land at the coast, dip poleward. Make poleward dips if it just came from a forest. Dip poleward for deserts, and dip equatorward if it just came out of a desert. Do not cross the equator into the other hemisphere. Dip equatorward for mountains and any glaciers. Lakes and wetlands get equatorward dips if crossed.
high and low pressure
  • Label high pressure if an area generally has jet streams circling clockwise in the northern hemisphere and counter-clockwise in the southern hemisphere. For low pressure, the jets should be circling counter-clockwise in the northern hemisphere and clockwise in the southern hemisphere. Label warm and cold fronts around the warm sectors of low pressure systems. The fronts can connect with other low pressure systems but not with high pressure. They should not cross the equator. Create isobars around the high and low pressure. Make the pressure systems especially intense if they are large. Have generally higher pressure at lower areas and lower pressure on mountains and high elevation, including ice as elevation. Make sure you do these for each season, as with below.
prevailing winds
  • Draw wind direction around the pressure systems. Remember which direction the pressure systems turn in. Wind also goes away form high pressure and toward low pressure. Label upwelling and sinking of wind, it rises if it hits mountain ranges, sinks if it hits a valley or the ocean from high elevations, rises at low pressure and sinks at high pressure. Also label upper-level winds which go from upwellings towards sinkings. Wind should be allowed to cross the equator.
ocean currents
  • Draw warm and cold ocean currents. Draw cold ocean currents in cold areas and where the wind goes equatorward. Draw warm ocean currents near tropical reigons and areas where wind goes poleward. Only count surface winds. Do not draw them for lakes and inland seas disconnected from main sea unless they are enormous and deep. Make sure the currents avoid land even if it means going in the opposite direction from prevaling winds. Warm and cold currents should be allowed to converge and cold ocean currents should cross the equator unless something blocks it such as a massive release of freshwater into the oceans. Only the largest rivers should be allowed to nudge existing currrents away from its mouth. Cold currents should not enter shallow tropical gulfs, bays, and seas, and warm currents should not cross polar seas. Currents should be allowed to directly cross under a pressure system if nessecary.
precipitation
  • label wet and dry zones on the map. Areas where wind goes from the unfrozen sea or lake to land should be labelled wet. If warm currents approach land then makes a turn, colour in the areas it was approaching before it turned as wet. Leeward sides of mountains should be labelled dry. Areas directly under high pressure should be labelled dry. When cold ocean currents approach land, label these as dry. If wind from a large already wet place goes into another place, label it wet, and if wind from a large dry place goes to another place, label it wet. Label polar glaciers as dry. The windward side of a mountain, if the wind is from the ocean or a lake or a wet place, it should be labelled wet. Wind going from the land to the sea should be labelled dry. Areas where the jet stream goes from the ocean to land should be labelled wet until it turns around a large high pressure system. Areas ahead of a cold or warm front should be labelled wet, the cold front area larger in summer and warm fromt larger in winter. If the area is especially close to the equator, both spring/autumns should be considered summer. If an upwelling of air occurs near both the sea and the land, that area should be labelled wet. If upper-level winds go from the sea at an upwelling of air towards the land at moderate or high elevations, those areas should be labelled wet. Extremely large supercontinents should be labelled dry near the middle. Create lines denoting areas of equal precipitation for the particular month. Adjust the amount or precipitation of areas between wet and dry based on the global wetness at the era.
temperatures
  • If there is a large polar ice sheet on land, they are always cold, but warmer in summer than in winter. In polar areas, ocean should generally be warmer than land. Dips towards the poles should be warm, dips equatorward should be cooler. Mountains and higher elevation should generally be colder than surroundings. Temperate wet areas should be cooler than surroundings in spring and summer, and warmer in autumn and winter. Tropical wet areas should be cooler, tropical warm areas warmer. High pressure should be cooler if cold and warmer if warm. Low pressure should be warmer if cold, cooler if warm. Downwind from leeward side of mountains should be warmer. Poleward surface winds should be warmer, equatorward cooler. Consider the average global temperatures during that era and which areas are at which temperatures, create isotherms accordingly.
ecosystems and biomes
  • The ecosystems should be drawn on one map. Base it similarly to the modified Koppen system. If the total rainfall is very low for the four seasons, label it desert. Slightly wetter areas semidesert. Polar reigons, if there are glaciers, label it ice, if it’s cold and dry all year, label it tundra. If it’s sometimes warm in polar reigons and moderate precipitation, label it boreal forest. If a tropical area is warm and wet, label it rainforest. Label monsoonal reigons accorgingly and also based on their total precipitation. Label deserts hot and cold. Keep doing this according to both the climate in the areas and the likely biomes in the area.
final thoughts

Well, what do you think? I once tried this method myself (but much, much more simplicified) on the current climate and the near future, and got similar results to the real climate. Does this work, and what are some of the potential modifications tha tmay need to be made? No this is not homework. What would one discover if they tried to base it on global warming of the future? Would northern Europe cool down? Amazonia turn to grassland? Acatama and Namib deserts turn to mediterranean climate? Central North America become desert? Sahel become desert? Lower Tibet become desert? Monsoonal India become drier? North America west coast become drier? California become hotter? Increased El Ninos? SAL across the Atlantic? Little change in Atlantic hurricane activity? Is this similar to some of the climate modeling simulations? Can this be used to predict past and future climates based on continental drift? Or do we have to implement what we already know for the Holocene for it to be accurate? Thanks. ~AH1(TCU) 00:25, 13 March 2008 (UTC)[reply]

non-medical placebos?

Hi. I know when you apply a treatment and convince the patient it will work and it works, that's a placebo, what what about non-medical, ie. the power of suggestion? Like for example, if you go into a dark house, and someone convinces you it's haunted, then suddenly you start experiencing signs of "ghosts" when it's really your mind playing tricks on you based on the suggestion, what is this effect called? Or, if you convince yourself you're sick, but you're really not, and you begin to experince what you think are "symptoms" of some sickness? Thanks. ~AH1(TCU) 00:29, 13 March 2008 (UTC)[reply]

Perhaps you are thinking of psychosomatic or conversion disorder --Omnipotence407 (talk) 00:49, 13 March 2008 (UTC)[reply]
See also: nocebo effect, suggestion, hypnotism.

"Random outbursts give people cancer"?!?

Hi. This is not a request for medical advice. Is this possible? Or was this a sarcastic joke or itself a random outburst? Or is it brain cancer? Is the joke about "if your hand is bigger than your head, you have cancer" solely in order to slam the victim's hand into their face? Or was this question a random outburst? Thanks. ~AH1(TCU) 00:44, 13 March 2008 (UTC)[reply]

Complete and utter hoax. Wisdom89 (T / C) 00:50, 13 March 2008 (UTC)[reply]
Maybe they meant random outbursts of gamma rays. And yes, the thing about your hand is so that you can hit someone on the nose. That's it. --98.217.18.109 (talk) 00:58, 13 March 2008 (UTC)[reply]

Goatse - how does he do that with his butt?

How did the Goatse man manage to get his rectum so insanely stretchy? Is there a likely medical explanation for it (I saw a guy who could stretch his skin about two feet from his body on tv once, maybe its the same thing), or was it just a "gift" he was born with? How do you go about discovering that you can do that anyway? --62.136.16.236 (talk) 00:58, 13 March 2008 (UTC)[reply]

Well, you can bet he probably started small and worked his way up. If you view the whole series (not recommended), you'll see that he inserts extremely large objects into his rear before doing the classic pose. That's got to help, if that's your goal. --98.217.18.109 (talk) 01:01, 13 March 2008 (UTC)[reply]
It's actually a martial art perfected by the Chinese monk Goa Tse. See here. bibliomaniac15 01:02, 13 March 2008 (UTC)[reply]
Eww!! That's uncyclopedia!!!!!!
As can be seen in slideshow form at http://www.goatsemarathon.com (obviously not worksafe). I've sometimes wondered if his internet antics (legendary they may be) have affected his continence. --Kurt Shaped Box (talk) 01:49, 13 March 2008 (UTC)[reply]
Note to self: When 98.217.18.109 says "not recommended", you don't need to check for yourself. ៛ Bielle (talk) 02:09, 13 March 2008 (UTC)[reply]
It's been years since I've seen it yet I can remember some of the scenes like it was yesterday. It's one of those visual memories you sort of resent having semi-permanently burned into your brain somewhere. Also, re: speculation about his health, I doubt that's a terribly good thing to do for your body and probably has a number of long-term side effects. That's not medical advice though I doubt any doctor would recommend doing that. --98.217.18.109 (talk) 04:38, 13 March 2008 (UTC)[reply]
Men who engage long-term in normal (well, normal for some) receptive anal intercourse (let alone grotesque activities such as fisting) often end up with continence problems because of the weakening of the muscles. What this guy has done to himself ... I'm lost for words. -- JackofOz (talk) 05:20, 13 March 2008 (UTC)[reply]
Jack, being familiar with all matters Down Under, speaks wisely. If this practice hasn't already wrecked him, I'm confident it soon will. I'd wager that traditional medicine would frown on this practice, but holistic medicine would open up to it, perhaps. StuRat (talk) 06:53, 13 March 2008 (UTC)[reply]
I dont know if the Down Under comment was accidental Stu, but its hilarious nevertheless! —Preceding unsigned comment added by 79.76.144.62 (talk) 01:44, 14 March 2008 (UTC)[reply]
Jack, would you be speaking from experience or is it just fistwishful thinking? —Preceding unsigned comment added by 79.76.144.62 (talk) 01:38, 14 March 2008 (UTC)[reply]
Might it cause a Fist ula? Edison (talk) 18:22, 13 March 2008 (UTC)[reply]
Sounds odd - but I think that the internet would probably be a poorer place without the Goatse Man. He's inspired a lot of people to do creative, humourous and culture jamming-related things over the years - just take a look at all those 'tribute to Goatse' and 'Goatse sightings in the real world' pages. Heh, Time Magazine may have even placed a deliberate nod to him on one of their covers... --Kurt Shaped Box (talk) 17:41, 13 March 2008 (UTC)[reply]
Most people believe the goatse man is Kirk Johnson. If you are sick enough to compare his vast collection of photos and videos to the official goatse photo, it is difficult to imagine that Kirk is not the goatse man. So, now that you have a good idea who he is, you search for his email address and ask him about his practices. Who knows, he may have written books or produced do-it-yourself videos. -- kainaw 18:54, 13 March 2008 (UTC)[reply]
I'm pretty sure the goatse man is dead. In fact just Google that phrase (goatse man is dead) and you'll find plenty of links, none of which I'm going to attempt to visit from my workplace computer. --LarryMac | Talk 19:11, 13 March 2008 (UTC)[reply]
If you heard a story about him dying at the age of 70-something, that was from a hoax newspaper article knocked up by the boys at Stileproject years ago. If he died recently, I haven't heard anything about it. --Kurt Shaped Box (talk) 22:50, 13 March 2008 (UTC)[reply]

psa levels

can alchohol consumption effect psa levels? —Preceding unsigned comment added by Jon julie (talkcontribs) 02:03, 13 March 2008 (UTC)[reply]

If people didn't drink so much, there wouldn't be so many public service announcements about drunk driving and so forth. I hope this answers your question.--The Fat Man Who Never Came Back (talk) 02:26, 13 March 2008 (UTC)[reply]
To translate, "psa" could refer to anything on this page. Could you please (a) work out which one you're talking about, (b) check the article about it to see if it mentions the effect of alcohol, and (c) if it doesn't, feel free to come back here and clarify what you meant. And the grammar freak in me would like to point out that the word you probably meant to use was affect, although context may prove effect correct. Confusing Manifestation(Say hi!) 02:46, 13 March 2008 (UTC)[reply]
I enjoyed my joke, but he's probably referring to Prostate specific antigen levels.--The Fat Man Who Never Came Back (talk) 02:48, 13 March 2008 (UTC)[reply]
I enjoyed it too, and I agree that that's the likely candidate, but you know what they say about people who assume. Confusing Manifestation(Say hi!) 02:50, 13 March 2008 (UTC)[reply]
Don’t assume that we know what they say. — Knowledge Seeker 09:34, 13 March 2008 (UTC)[reply]
Assume--Shniken1 (talk) 10:16, 13 March 2008 (UTC)[reply]

About Albert Einstien (E=mc^2)

On deriving E=mc², did Einstien also come to know how to travel with the speed of light by converting mass into energy (energy in the form of electromagnetic radiation) and back to mass? i.e, did he know how to travel so fast that one could almost 'dissappear' from one place and 'appear' at another place almost at the same time?59.95.77.132 (talk) 05:34, 13 March 2008 (UTC)[reply]

The short answer is no, certainly not for "everyday" objects - there is usually an irreversible increase in entropy when mass is converted to energy. But see our article on teleportation for recent developments. Gandalf61 (talk) 07:17, 13 March 2008 (UTC)[reply]
E=mc² wasn't a process for extracting energy, it was an upper bound on the energy that might be extracted by any process. Prior to this the only way you could quantify energy was by the amount converted from one form to another. You could extract work from a substance by converting it into something else, but (aside from thermodynamic limitations) it was never clear whether you'd gotten all the work you could get out of it or whether there was some yet-undiscovered way of getting more. Einstein's argument suggested that an object of (inertial) mass m had a total of mc² energy, meaning that that was the most you could ever extract—after that there would be nothing left of the original object. It said nothing about whether there was actually any physical process that could extract that much energy. However at the time people had already noticed that radioactive substances seemed to release an enormous amount of energy—though very slowly—and I think people imagined or hoped that by studying and harnessing this process they could get the full mc² energy out of ordinary matter. It's now known that this isn't correct—radioactivity is just another kind of chemical process, involving nuclear bonds instead of atomic bonds, and just rearranges the constituents of matter without creating or destroying them—but nuclear energy seems to be permanently associated with E=mc² in the public consciousness. It's also now known that you can't turn the basic constituents of matter into useful energy without violating various conservation laws. Actually all of the laws that you need to violate seem to be only approximate, but the situations in which they're violated are pretty extreme, and it's very hard for me to believe that you could ever construct a useful (and safe!) power source this way. You can get plenty of energy from nuclear chemistry alone, and for that you just need the laws of nuclear chemistry, not E=mc². So although E=mc² is of major theoretical importance, it's not relevant to any practical technology. -- BenRG (talk) 13:44, 13 March 2008 (UTC)[reply]
Nuclear fission and radioactivity are not chemical processes. Nuclear chemistry covers different aspects of a nuclear power plant to the actual energy production. AlmostReadytoFly (talk) 14:48, 13 March 2008 (UTC)[reply]
Furthermore, massive particles such as beta particles, neutrinos and antineutrinos are created in radioactive decay. AlmostReadytoFly (talk) 15:02, 13 March 2008 (UTC)[reply]
And, uh, E=mc^2 is relevant to practical technology. There is a real mass differential between U-235 and its fission products, and the energy resulting from the fission reaction (mostly in the form of the kinetic energy of the repelling fission pieces) corresponds exactly to that "missing mass" times the speed of light squared. It comes into play in fusion reactions as well. To say nothing of atomic weapons! It's not an accident or an error that the public associate E=mc^2 with nuclear energy. Fission doesn't violate any conservation laws—total energy of the system is conserved even if some of it is in the form of nuclear bonds and then it becomes kinetic energy, etc. It does violate conservation of mass, but all of SR does that, and that's OK. --98.217.18.109 (talk) 22:14, 13 March 2008 (UTC)[reply]

Tell me if it' snot a good idea....

...to make an air filter based on the design of the human nose. That is, it would consist of several long tubes filled with fibers (nose hairs) extending from the walls, coated with a thick fluid dripping from the walls onto the fibers. Air would be blown through the tubes. The nostrils would need to be occasionally cleaned, perhaps with a silicone corkscrew that could be run through them once a day. Some type of antibacterial solution would need to be used in the drip, to prevent nasties from growing in it. StuRat (talk) 06:39, 13 March 2008 (UTC)[reply]

Snot for me my friend. Who gets to work the corkscrew? Julia Rossi (talk) 07:26, 13 March 2008 (UTC)[reply]
I suppose the high-end models could have this function automated. On the other hand, if the liquid drips through quickly enough, that may clean the nostrils out sufficiently without needing a corkscrew. StuRat (talk) 14:09, 13 March 2008 (UTC)[reply]
Underemployed telephone sanitisers.
Atlant (talk) 13:10, 13 March 2008 (UTC)[reply]
The nose is not designed primarily as a filter, and it doesn't make a particularly good one. I don't think we're gonna see this on Dragons' Den in the near future.--Shantavira|feed me 08:33, 13 March 2008 (UTC)[reply]
Citation please? Seems to me that noses are excellent filters. I don't know about you but whenever I'm inside where the air quality is lower, I'm breathing through my nose to filter our some of the dust etc. And yes the nose is of course an excellent olefactory device, but if I were to posit its main functions, smell would be 1A and getting filtered oxygen to the lungs would be 1B. Vranak (talk) 14:40, 13 March 2008 (UTC)[reply]

This is actually done. Many mechanical air filters are coated with a water-soluable "stickum". They then server their purpose for a while, get dirty, are washed up, recoated, and placed back in service for another cycle. Light oil is also used but requires a detergent or solvent during the washing-up phase. My sports car has just such a filter.

Atlant (talk) 13:12, 13 March 2008 (UTC)[reply]

This wouldn't be much different than blowing air past one of those hanging fly-strips and hoping all the nasties get stuck to it. They don't. You have two things working against you. You need an adhesive that will trap everything - which will be difficult to find. The adhesive cannot dry out as air blows past - which will be difficult to find. -- kainaw 13:16, 13 March 2008 (UTC)[reply]
I don't think any filter removes 100% of the particulates from the air with each pass. As for drying out, my idea was to have a reservoir of liquid which slowly drips over the fibers. To make the fluid last longer, it should be oil-based instead of water-based. Glue traps for mice use such an adhesive, and they stay tacky for quite a long time. To do double duty, the dripping oil could have a nice scent to it, like cinnamon or mint, to provide "aromatherapy" at the same time it cleans the air. StuRat (talk) 14:00, 13 March 2008 (UTC)[reply]
This makes me wonder about bubbling air up through a reservoir of sticky oil. By the time the bubble hits the top, much of the gunk will get stuck. No need to clean anything. Just dump out the reservoir and refill as necessary. -- kainaw 15:03, 13 March 2008 (UTC)[reply]
That might work, or it might just make a thick foam that fills the entire house. :-) StuRat (talk) 15:29, 13 March 2008 (UTC)[reply]
Isn't Kainaw's idea simular to how a bong works? However, those use water and get gunky relatively quickly. Some of our inventions based off nature work quite well (velcro) while others not so well (Ornithopter). But as for this invention...who nose? 206.252.74.48 (talk) 15:40, 13 March 2008 (UTC)[reply]
The difference is that bubbles containing water quickly pop as the water evaporates, but this is not so for bubbles made of oil. StuRat (talk) 16:46, 13 March 2008 (UTC)[reply]
Congratulations you’ve made it onto my list. --S.dedalus (talk) 22:44, 13 March 2008 (UTC)[reply]

volume of space

may we say that the space has afinite volume but infinite surface,like Gabriel`s horn???thank youHusseinshimaljasimdini (talk) 11:18, 13 March 2008 (UTC)[reply]

You can say anything you like. It's a free Internet. However, there are existing arguments about what constitutes the "end" of space. Is it infinite? Does it have a boundary? Does it wrap back upon itself? -- kainaw 12:10, 13 March 2008 (UTC)[reply]
If space has infinite volume I would surmise that it has infinite dimension (radius?) and so would come to the conclusion that the outer surface would have infinte area. Nevertheless the concept of an outer surface of infinite radius is a difficult one to consider.87.102.94.198 (talk) 12:51, 13 March 2008 (UTC)[reply]
The general belief is that physical space doesn't have a boundary at all. -- BenRG (talk) 12:56, 13 March 2008 (UTC)[reply]
Husseinshimaljasimdini - I am not clear what you mean by "infinite surface". You seem to be assuming that the Universe has a boundary. As BenRG says, all common models for the topology of the Universe assume that the Universe does not have a boundary (don't be misled by the use of the word "horn" in models such as the Picard horn). On the other hand, you say that "space has a finite volume". Some observations of possible periodicities in the cosmic microwave background hint at a finite universe but this is not certain, and there are still plausible models in which the Universe is infinite. Possibly when you say "space" you may be thinking of the observable universe (which is finite) rather than the whole Universe ? Our article on the shape of the Universe has more details. Gandalf61 (talk) 13:08, 13 March 2008 (UTC)[reply]

buisness

what is the importance of buisness to the society?nipet —Preceding unsigned comment added by Nipet (talkcontribs) 12:15, 13 March 2008 (UTC)[reply]

Take a look at The Wealth of Nations. It will point you in the right direction. – ClockworkSoul 13:28, 13 March 2008 (UTC)[reply]

Gases "dissolved" in liquids; fish breathing

Can someone explain to me, in layman's terms, how exactly oxygen is dissolved in sea water and how fish breathe with their gills? Is the oxygen in sea water in the form of tiny bubbles, which the fish's gills suck into their bloodstream in the same way that land animals absorb oxygen through their lungs? Or is the oxygen somehow chemically bound to water or other molecules in sea water as it is to hemoglobin in the blood, in which case the fish's gills would presumably perform a chemical reaction? Or is there some other mechanism involved? Thanks! Marco polo (talk) 13:20, 13 March 2008 (UTC)[reply]

Refer to solubility. To put it in very simple terms, consider water as being composed of many small particles (molecules) of water. These particles nonetheless have a lot of space between them, and molecules of oxygen are able to fill some of that empty space. This is the oxygen dissolving in the water. The way the oxygen then gets transferred into a fish's bloodstream is a similar process to how lungs work. Water is passed over the gills, and the oxygen diffuses into the blood where it is at a lower concentration than in the water. It doesn't require a chemical reaction on the gills to extract the oxygen. --jjron (talk) 14:11, 13 March 2008 (UTC)[reply]
Nitpick: diffusion across a cell membrane, in this case that of the gill's cells, is called osmosis. --Bowlhover 15:04, 13 March 2008 (UTC)

Well water is made partially of oxygen (It's H2O) but I don't think that is how they do it. I'm sure there is oxygen that is disolved in the water (not in the form of bubbles otherwise fish could breath out of water) but liquid, if that makes sense. I'm not sure how this happens since oxygen has to be REALLY cold before it becomes a liquid on its own. I wouldn't mind hearing an answer to this also. EDIT: Oh hey an answer TheGreatZorko (talk) 14:15, 13 March 2008 (UTC)[reply]

jjron's answer covers that aspect fairly well. As for how the air is initially dissolved in water, there are several mechanisms. Wave action at the surface forces tiny bubbles into the water, which then dissolve into the water. Plants in the water also give off oxygen as a result of photosynthesis, which is then dissolved in the water. Even still water with no plant life will absorb some oxygen at the surface. Ocean currents are critical to the distribution of this dissolved oxygen, which otherwise would be at a much higher concentration near the surface than deep underwater. Note that other gases in air, like nitrogen and carbon dioxide, also dissolve in water. There is an occasional problem where an area of water is low on oxygen, creating a dead zone. Aquariums which lack wave action or sufficient plants often add oxygen to the water by bubbling air through it. StuRat (talk) 14:29, 13 March 2008 (UTC)[reply]
oxygen has to be REALLY cold before it becomes a liquid on its own
Doesn't matter. Salt has to be REALLY hot before it melts into a liquid on its own, but you have no problem with salt dissolving in water, right? —Keenan Pepper 14:49, 13 March 2008 (UTC)[reply]
Check out Lake Nyos for an interesting gas-dissolved-in-water story. --Sean 14:51, 13 March 2008 (UTC)[reply]
I've looked at the Lake Nyos article several times before, and I can't help but wonder at the thought processes and value systems I see. I'd support the "lower the level" plan a lot more cheerfully if all of the planners' children lived downstream. Lowering the pressure, without reducing the gas load, seems, well, unwise. I mean, they already have a vent pipe which works on it's own, powered solely by the rising gases coming out of solution. The vent pretty much proves that we've figured out what we're dealing with. So, given that kind of supersaturation, what happens to the immense pressure in the lower levels if the water level is lowered by the proposed 20m? You are dropping the pressure by about 5 atmospheres, which will, incidentally, allow some of the dissolved gas down there to come out on it's own, and the rising bubbles will probably cause the impending-disaster-to-be-prevented to happen right then and there. I know, take it to the discussion page. -SandyJax (talk) 18:45, 13 March 2008 (UTC)[reply]
The "lower the level" plan has nothing to do with dissolved gas. It's about preventing a landslide and resulting flood that could kill tens of thousands. --Carnildo (talk) 21:56, 13 March 2008 (UTC)[reply]
I saw a movie where a pressurised water suit had a large amount of oxygen disolved in it. Can you breath water if enough oxygen is disolved in it?--155.144.251.120 (talk) 03:32, 14 March 2008 (UTC)[reply]

International Space Station

So far the international space station has always housed 3 crew members. When the ISS is finished, will it house more crew members. (I don't believe our article says; I read through it but it is possible I missed it). ike9898 (talk) 16:26, 13 March 2008 (UTC)[reply]

According to this SPACE.com article, the final crew capacity will be six or seven people. --Bowlhover 21:27, 13 March 2008 (UTC)

IV nurse (medicine)

A question from Germany: What is an "IV nurse"? I have heared it in Grey's Anatomy ("IV nurse to ICU") What is "LR"? - a medical acronym. ("two liter LR running wide open") (Kind of a liquid for i.v. application to substitute a volume deficit.) --84.137.46.213 (talk) 20:28, 13 March 2008 (UTC)[reply]

Isn't a IV nurse a nurse who knows how to do intravenous therapy? --Bowlhover 21:18, 13 March 2008 (UTC)
and also assigned to carry out IV therapy, as opposed to other duties. (If five nurses are on a single hospital floor, they may all know how to do IV therapy, but only one might be assigned to be the IV nurse). - Nunh-huh 02:02, 14 March 2008 (UTC)[reply]
LR is lactated Ringer's solution. TenOfAllTrades(talk) 21:28, 13 March 2008 (UTC)[reply]
Thanks for the fast answers. --84.137.46.213 (talk) 21:45, 13 March 2008 (UTC)[reply]

What are the night-blooming vegetables?

We are interested in a comprehensive list of night-blooming vegetables. —Preceding unsigned comment added by Webstergl (talkcontribs) 21:09, 13 March 2008 (UTC)[reply]

If you include herbs, you might like to look into these: Silver Thyme, 'Alba' or white Eggplant, white pumpkins, Basil, Mint, Oregano according to[20]. Silver or reflective foliage and white fruits seem to be indicators. Bats and moths the agents. Are you thinking of starting an article here? hint hintJulia Rossi (talk) 23:29, 13 March 2008 (UTC)[reply]

if cocaine is just a molecule, why dont peeple synthesize it?

so if cocaine is just a molecule (picture to the right in that article) why don't peeple synthesize it, like they do with meth or lsd, which are also molecules right? why go thru all the trouble of stuffing it up some immigrants butt excuse my french to get it past border control thank you —Preceding unsigned comment added by 79.122.103.78 (talk) 22:10, 13 March 2008 (UTC)[reply]

Because synthesizing it takes time, effort, and some technical know how. Wisdom89 (T / C) 22:18, 13 March 2008 (UTC)[reply]
Even LSD and methamphetamine have starting materials, and those happen to be quite readily available. But as mentioned in Cocain#Synthesis, creating a drug from scratch would be difficult, problematic, and also very expensive. The only answer I can give you is to guess that unlike the other two drugs, there are simply no possible starting materials that occur naturally (or in pharmacies) in sufficient quantity to make synthesis cheaper than smuggling. Someguy1221 (talk) 22:18, 13 March 2008 (UTC)[reply]
(ec) Well, sneaking it past border patrol is actually pretty cost-effective for cocaine, which has a very high street value for its relatively small volumes, unlike LSD and methamphetamines (cocaine costs roughly 10X that for the same volume of meth, if I recall). So you only have to smuggle a small amount to make up for the price of smuggling and the dangers associated. In any case, there is a section on synthesis in the article: Cocaine#Synthesis. It's not cited but it sounds plausible to me: "Synthetic cocaine would be highly desirable to the illegal drug industry, as it would eliminate the high visibility and low reliability of offshore sources and international smuggling, replacing them with clandestine domestic laboratories, as are common for illicit methamphetamine. However, natural cocaine remains the lowest cost and highest quality supply of cocaine. Actual full synthesis of cocaine is rarely done. Formation of inactive enantiomers and synthetic by-products limits the yield and purity."--98.217.18.109 (talk) 22:20, 13 March 2008 (UTC)[reply]
(Of course, I am aware that the cost of the drug is directly related to the availability. But I'm just pointing out that when the price is that high the smuggling isn't all that bad a method, even though, of course, the price is that high in part because smuggling is the method.) --98.217.18.109 (talk) 22:22, 13 March 2008 (UTC)[reply]
The answer to these types of questions is almost always "Because it's not cost effective". If people could make big money synthesizing cocaine, they would presumably already be doing it. Friday (talk) 23:38, 13 March 2008 (UTC)[reply]
An important part of the answer is that setting up a cocaine-synthesis plant would require a lot of up-front investment for an uncertain return, while paying somebody to smuggle it doesn't require much investment at all. --69.134.115.242 (talk) 00:05, 14 March 2008 (UTC)[reply]

March 14

physic

richard fenyman —Preceding unsigned comment added by 71.10.27.69 (talk) 00:14, 14 March 2008 (UTC)[reply]

We just happen to have an article on Richard Feynman, if that's who you're looking for.--VectorPotentialTalk 00:17, 14 March 2008 (UTC)[reply]

Quantum Physics

Why does each atom have different allowed energy levels for its electrons? I understand that each electron is actually a "particle-wave" and its orbits are only the ones allowed so the electron-wave won't cancel itself. So, why are there different orbits allowed for each compound and atom? Every electron is the same so how are there different orbits in each atom that don't make them cancel? For example, in one atom the orbits may be consecutively 2eV then 5eV, but in another, there may be an allowed orbit of 3.5eV where in the other atom this would not have been allowed. Why is this? Many thanks, Zrs 12 (talk) 00:18, 14 March 2008 (UTC)[reply]

The article on atomic orbitals will give you the full answer on this. However, my simplified answer to this is that the charge in the nucleus is different for each type of atom. Consequently, the electric potential energy is different in different atoms. The full quantum mechanical solution for atoms more complex than helium is not trivial and cannot be expressed as a neat exact mathematical formula. Numerical solution techniques must be used. For hydrogen, an exact expression is possible as derived in the Bohr model but this relies on simplifications that cannot be applied to more complex atoms. SpinningSpark 00:47, 14 March 2008 (UTC)[reply]
Mathematical side note: Roughly speaking, the calculation of energy levels involves solving the time-independent Schrödinger equation
where is the energy to be solved, is the potential energy of the electron. The depends on different kind of atom since is the integral of electrostatic force experienced by the electron and is affected by the proton number in the nucleus of different kind of atoms. Therefore, the difference in makes different set of energy levels solved. - Justin545 (talk) 02:05, 14 March 2008 (UTC)[reply]
And atoms with multiple electrons, have the electrons influencing each other. The Pauli exclusion principle applies as electrons are fermions. It forces electrons to adopt different positions in the atom. Graeme Bartlett (talk) 00:53, 14 March 2008 (UTC)[reply]
For a very simplified explanation, see the energy levels article SpinningSpark 01:15, 14 March 2008 (UTC)[reply]

What species of seal or walrus is this?

http://www.youtube.com/watch?v=DDg7kWgs5e0 --Sonjaaa (talk) 02:17, 14 March 2008 (UTC)[reply]

RL circuit

I am working on a physics laboratory project dealing with an RL circuit (the resistor and inductor are in series with a battery). I am trying to find out the work done by the battery over a period of time from when the circuit is connected, and the energy dissipated from the resistor in that same time period. I realize that to find the work done I need to integrate the expression for the power (P=IV) over time, but I'm not sure how I ought to go about doing that. —Preceding unsigned comment added by 134.129.115.87 (talk) 02:19, 14 March 2008 (UTC)[reply]

The work done by the battery is equal to the energy dissipated in the resistor. Is that any help? —Preceding unsigned comment added by SpingMander (talkcontribs) 02:36, 14 March 2008 (UTC)[reply]